Med-Surg/Adult Health Qs

Pataasin ang iyong marka sa homework at exams ngayon gamit ang Quizwiz!

Which significant risk factors for coronary heart disease carry a greater risk for women than for men? Select all that apply. 1. Obesity 2. Smoking 3. Hypertension 4. Diabetes mellitus 5. Low levels of high-density lipoprotein (HDL) cholesterol

4, 5 Diabetes is twice as strong a predictor of coronary heart disease in women as in men; diabetes cancels the cardiac protection that estrogen provides to premenopausal women. A low level of HDL-C (less than 35 mg/dL) has a greater bearing on coronary heart disease in women than in men and is the most important lipid factor in women; the significance of this is unclear. Obesity, smoking, and hypertension are risk factors common to both women and men.

A health care provider prescribes an antidysrhythmic to be administered at 1000 mcg intravenously (IV) per minute. The directions state that 500 mg of the drug should be added to 500 mL of D5W. At what rate should the nurse set the volume control device to administer the medication correctly? Record the answer using a whole number. __ mL/hr

60 1000 mcg = 1 mg; 500 mg of the drug added to 500 mL of IV fluid results in a solution in which 1 mL contains 1 mg of the drug. The prescription is for 1 mg/min. The volume control device delivers milliliters per hour; therefore, the device should be set at 60 mL/hr.

The nurse provides discharge medication education to a client who has been switched from a prescription for heparin to a prescription for warfarin sodium (Coumadin). The nurse concludes that the teaching was effective when the client states, "I will: 1. ... take acetaminophen (Tylenol) for my occasional headaches." 2. ... spend most of the day working at my desk." 3. ... ask my health care provider for antibiotics before going to the dentist." 4. ... make an appointment to have a complete blood count drawn."

1 Acetaminophen should be used when an analgesic is required because it does not interfere with platelet aggregation. Acetylsalicylic acid (aspirin) should be avoided because it interferes with platelet aggregation. Immobility causes venous pooling and can predispose the client to deep vein thrombosis. Antibiotics are not necessary when going to the dentist; this is done when clients have cardiac problems, such as rheumatic fever or cardiac surgery. A prothrombin time (PT) or international normalized ratio (INR), not a complete blood count, needs to be done periodically.

A client is found unconscious and unresponsive. What should the nurse do first? 1. Initiate a code. 2. Check for a radial pulse. 3. Compress the lower sternum. 4. Give four full lung inflations.

1 Additional help and a cardiac defibrillator must be obtained immediately. The carotid, not radial, pulse is used. Compressing the lower sternum is done after the nurse summons help. Two lung inflations are given after 30 chest compressions.

A nurse is performing an admission health history and physical assessment of a client with thyroid problems. What information is most important to obtain? 1. Heart rate and rhythm 2. Appetite and condition of the skin 3. Respiratory rate and depth of inhalations 4. Intolerance to heat and decrease in weight

1 Because of the increased metabolic rate associated with excess thyroid hormones, the heart rate increases; the client is at risk for tachycardia, palpitations, dysrhythmias (e.g., atrial fibrillation), angina, and heart failure. Although the client probably will have an increased appetite and moist, warm skin, these physiological responses that are related to the increased metabolic rate are not life threatening. Although the client's respiratory rate will increase, the depth of respirations may or may not increase; these physiological responses are related to the increased metabolic rate and are not life threatening. Although the client will be intolerant to heat and lose weight because of the increased metabolic rate, these physiological responses are not life threatening.

A client with a history of closed-angle glaucoma is scheduled for abdominal surgery. Because the client is extremely anxious, surgery is to be performed under general anesthesia. What should the nurse teach the client to do to prevent respiratory complications postoperatively? 1. Deep-breathing techniques 2. Performing productive coughing 3. Turning from side to side frequently 4. Pant breathing while gently closing the eyelids

1 Deep breathing is an intervention to prevent respiratory complications that does not increase intraocular pressure. Coughing is contraindicated because it increases intraocular pressure. Although turning from side to side is permitted, it is not as effective as deep breathing in preventing respiratory complications. Pant breathing is shallow breathing and will not prevent respiratory complications.

A client returns from a bronchoscopy, and the nurse provides the client with instructions to not consume any food or drink any fluids for several hours. The nurse explains that these measures are taken to prevent: 1. Aspiration 2. Dysphasia 3. Projectile vomiting 4. Abdominal distention

1 To allow for the insertion of the bronchoscope, throat muscles are anesthetized, diminishing the protective gag reflex. Dysphasia is difficulty in talking and does not occur with a bronchoscopy. Projectile vomiting does not occur after a bronchoscopy. A general anesthetic usually is not used; therefore, paralytic ileus is not a complication.

A client with heart failure is receiving digoxin (Lanoxin) and hydrochlorothiazide (HydroDIURIL). The nurse will assess for which signs and symptoms of digoxin toxicity? Select all that apply. 1. Nausea 2. Yellow vision 3. Irregular pulse 4. Increased urine output 5. Pulse rate of 64 beats per minute

1, 2, 3 Signs and symptoms of digoxin toxicity include: bradycardia, headache, dizziness, confusion, nausea, and visual disturbances (blurred vision or yellow vision). In addition, ECG findings may include heart block, atrial tachycardia with block, or ventricular dysrhythmias, all causing an irregular pulse. Increased urine output is an expected effect of the diuretic furosemide; a pulse rate of 64 beats per minute is an acceptable rate when a client is receiving digoxin.

The nurse recognizes that a common conflict experienced by the older adult is the conflict between: 1. Youth and old age 2. Retirement and work 3. Independence and dependence 4. Wishing to die and wishing to live

3 A common conflict confronting the older adult is between the desire to be taken care of by others and the desire to be in charge of one's own destiny. The conflict between the young and old age may occur but is not common. The conflict between the retired and working may occur but is not common. The conflict between those wishing to die and those wishing to live may occur but is not common.

A client with a history of dysrhythmias is to wear a Holter monitor for 24 hours on an outpatient basis. What should the nurse teach the client to do while wearing the monitor? 1. Discontinue medications. 2. Avoid using a microwave oven. 3. Keep a written account of activities. 4. Record the blood pressure periodically.

3 A day's activities should be recorded so that they can be compared with the occurrence of dysrhythmias. It is not necessary to discontinue medications. It is not necessary to avoid using a microwave oven. Obtaining blood pressure readings while wearing a Holter monitor is not necessary.

Tissue plasminogen activator (t-PA) is to be administered to a client in the emergency department. Which is the priority nursing assessment? 1. Apical pulse rate 2. Electrolyte levels 3. Signs of bleeding 4. Tissue compatibility

3 Assessment for bleeding is a priority when administering a thrombolytic agent because it may lead to hemorrhage. The heart rate is not affected. Electrolyte levels are not affected. Tissue compatibility is not necessary.

A nurse provides teaching regarding vitamin B12 injections to a client with pernicious anemia. The nurse concludes that the teaching was understood when the client states, "I must take the drug: 1. When feeling fatigued." 2 Until my symptoms subside." 3 Monthly, for the rest of my life." 4 During exacerbations of anemia."

3 Because the intrinsic factor does not return to gastric secretions even with therapy, B12 injections will be required for the remainder of the client's life. The drug must be taken on a regular basis for the rest of the client's life.

A client is admitted to the hospital with a diagnosis of laryngeal cancer. What is a common early sign of laryngeal cancer for which the nurse should assess this client? 1. Aphasia 2. Dyspnea 3. Dysphagia 4. Hoarseness

4 Hoarseness is caused by the inability of the vocal cords to move adequately during speech when a tumor exists. Aphasia refers to an expressive or receptive communication deficit as a result of cerebral disease; it is not related to laryngeal cancer. Dyspnea is a late, not early, adaptation that occurs with laryngeal cancer when a tumor is large enough to obstruct air flow. Dysphagia is a late, not early, adaptation that occurs when the tumor is large enough to compress the esophagus.

A client with a long history of bilateral varicose veins questions a nurse about the brownish discoloration of the skin on the lower extremities. What should the nurse include in the response to the client's question? 1. The arterial blood supply is inadequate. 2. There is delayed healing in the area after an injury. 3. The production of melanin in the area has increased. 4. There is leakage of red blood cells (RBCs) through the vascular wall.

4 Increased venous pressure alters the permeability of the veins, allowing extravasation of RBCs; lysis of RBCs causes brownish discoloration of the skin. Varicose veins do not affect the arterial circulation. Although healing may be delayed, the brownish discoloration does not result from trauma. There is no increase in melanocyte activity in individuals with varicose veins.

When administering albumin intravenously, the nurse considers that body water will shift from the: 1. Interstitial compartment to the extracellular compartment 2. Intravascular compartment to the interstitial compartment 3. Extracellular compartment to the intracellular compartment 4. Intracellular compartment to the intravascular compartment

4 Intravenous albumin increases colloid osmotic pressure, resulting in a pull of fluid from the interstitial and intracellular compartments to the intravascular compartment. The interstitial compartment is part of the extracellular compartment. Intravascular compartment to the interstitial compartment and extracellular compartment to the intracellular compartment are opposite to the actual shift of fluids when albumin is administered.

A client who is receiving chemotherapy for lung cancer has nausea and vomiting because of the therapy. The client wants to know if it is true that smoking marijuana will help. What is the nurse's best response? 1. "Smoking marijuana is not legal in any state." 2. "Marijuana is effective for nausea and vomiting if it is injected." 3. "Marijuana is not proven to be effective in preventing chemotherapy-induced nausea and vomiting." 4. "Tetrahydrocannabinol is an ingredient in marijuana that acts as an antiemetic in some people."

4 Tetrahydrocannabinol, an ingredient in marijuana, acts as an antiemetic in some persons and can be absorbed through the gastrointestinal tract or inhaled. The statement "Smoking marijuana is not legal in any state" does not answer the client's question. Marijuana is not injected. Tetrahydrocannabinol, an ingredient in marijuana, is an effective antiemetic for some clients.

When interpreting an ECG rhythm strip, the nurse identifies that ventricular contraction is displayed as the: 1. P wave 2. T wave 3. PR interval 4. QRS interval

4 The QRS interval represents time taken for depolarization of both ventricles. The P wave represents repolarization of the atria. The T wave represents repolarization of the ventricles. The PR interval represents the time taken for the impulse to spread through the atria.

Which client assessment finding should the nurse document as subjective data? 1. B/P 120/82 2. Pain rating of five (5) 3. Potassium 4.0 mEq 4. Pulse oximetry reading of 96%

2 Subjective data is obtained directly from a client. Subjective data is often recorded as direct quotations that reflect the client's feelings about a situation. Vital signs, laboratory results, and pulse oximetry are examples of objective data.

A nurse provides dietary instruction to a client who has iron deficiency anemia. Which food choices by the client does the nurse consider most desirable? Select all that apply. 1. Raw carrots 2. Boiled spinach 3. Sweet potatoes 4. Brussels sprouts 5. Asparagus spears

2, 3 One cup of boiled spinach contains 6.42 mg of iron. One cup of mashed sweet potatoes contains 3.4 mg of iron. One cup of cut carrots contains 1 mg of iron. One cup of Brussels sprouts contains 1.1 mg of iron. One cup of cut asparagus contains 1.2 mg of iron.

A client's laboratory report indicates the presence of hypokalemia. For which clinical manifestations associated with hypokalemia should the nurse assess the client? Select all that apply. 1. Thirst 2. Anorexia 3. Leg cramps 4. Rapid, thready pulse 5. Dry mucous membranes

2, 3 The gastrointestinal manifestations associated with hypokalemia are caused by decreased neuromuscular irritability of the gastrointestinal tract; this results in anorexia, nausea, vomiting, and decreased peristalsis. Because of potassium's role in the sodium-potassium pump, hypokalemia results in altered neuromuscular functioning, which precipitates leg cramps. Thirst is associated with hypernatremia. Rapid, thready pulse is associated with dehydration and hyponatremia. Dry mucous membranes are associated with hypernatremia.

A nurse assesses a client's intravenous site. What clinical finding leads the nurse to conclude that the intravenous (IV) site has been infiltrated? Select all that apply. 1. Redness along the vein 2. Coolness of skin near the insertion site 3. Swelling around the insertion site 4. Cessation in flow of solution 5. Vein feels hard and cordlike

2, 3, 4 When an IV infiltrates, the IV solution entering the interstitial space is at room temperature (approximately 75° F), whereas body temperature is approximately 98.6° F; therefore, the client's skin will feel cool to the touch at the site of an IV infiltration. In addition, the fluid in the interstitial space causes swelling around the insertion site, and the solution stops flowing. Redness along the vein, with the vein feeling hard and cordlike, is present with phlebitis.

The nurse who is working during the 8 AM to 4 PM shift must document a client's fluid intake and output. An IV is infusing at 50 mL per hour. The client drinks 4 ounces of orange juice and 6 ounces of tea at 8:30 AM, and vomits 200 mL at 9:00 AM. At 10:00 AM the client drinks 60 mL of water with medications and voids 550 mL of urine at 11:00 AM. At 12:30 PM, 3 ounces of soup and 4 ounces of ice cream are ingested. The client voids 450 mL at 2:00 PM. Calculate the total intake for the 8 AM to 4 PM shift. Record your answer using a whole number. ___ mL

970 mL 1 ounce = 30 mL; 1 cc =1 mL. 120 mL of orange juice at 8:30 AM. 180 mL of tea at 8:30 AM. 60 mL of water with medications at 10:00 AM. 90 mL of soup at 12:30 AM. 120 mL of ice cream at 12:30 AM (counted as a liquid because it melts at room temperature). 400 mL of IV fluid (50 mL × 8 hours = 400).

A nurse is providing dietary instruction to a client with cardiovascular disease. Which dietary selection by the client indicates the need for further instruction? 1. Whole milk with oatmeal 2. Garden salad with olive oil 3. Tuna fish with a small apple 4. Soluble fiber cereal with skim milk

1 Although oatmeal is a soluble fiber, whole milk is high in saturated fat and should be avoided. Olive oil contains unsaturated fat. Most fish have a low fat content; fruit does not contain fat. Soluble fiber helps to lower cholesterol; skim milk does not contain fat.

A nurse is providing postprocedure care to a client who had a cardiac catheterization via a brachial artery. For the first hour after the procedure, what is the priority nursing intervention? 1. Monitor the vital signs every 15 minutes 2 Maintain the client in the supine position 3 Keep the client's lower extremities in extension 4 Administer the prescribed oxygen at 4 L/min via nasal cannula

1 A cardiac catheterization may cause cardiac irritability; therefore, the client's vital signs should be monitored every 15 minutes for one hour and then every 30 minutes for the next two hours until stable. The vital signs may then be monitored every four hours. When a brachial artery is used for catheter insertion a low-Fowler, not supine, position usually is recommended because it promotes respirations. Keeping the client's lower extremities in extension is not necessary. A brachial, not femoral, artery was used for the catheter insertion. Although administering the prescribed oxygen at 4 L/min via nasal cannula may be done, it is not the priority. The client's response to the procedure is the priority.

The nurse expects that the most definitive test to confirm a diagnosis of multiple myeloma is: 1. Bone marrow biopsy 2. Serum test for hypercalcemia 3. Urine test for Bence Jones protein 4. X-ray films of the ribs, spine, and skull

1 A definite confirmation of multiple myeloma can be made only through a bone marrow biopsy; this is a plasma cell malignancy with widespread bone destruction. Although calcium is lost from bone tissue and hypercalcemia results, this is not a confirmation of the disease. Although Bence Jones protein is found in the urine, it does not confirm the disease. X-ray films will show the characteristic "punched-out" areas caused by the increased number of plasma cells, which contributes to the making of the diagnosis. The definitive diagnosis is made on biopsy.

A 78-year-old client comes to the health clinic complaining of fatigue. The client's laboratory results indicate a hematocrit of 32.1% and a hemoglobin of 10.5 g/dL. Which is the most appropriate nursing intervention in response to these laboratory results? 1. Conduct a complete nutritional assessment of the client. 2. Advise the client to come back to the clinic to have the test repeated in three months. 3. Nothing, because these are expected values for this age adult. 4. Investigate the cause of the anemia while understanding that mild anemia is an expected response to the aging process.

1 A nutritional assessment starts the investigation for a cause of the client's anemia and is an independent function of the nurse. Medical treatment should be initiated first and then the test should be repeated to determine the client's response to therapy; it is not within the legal function of the nurse to give medical advice. These are not expected values; an intervention is indicated. Anemia is not an expected response to the aging process.

A serious train accident occurs in the community. At the scene of the accident a triage nurse is identifying and labeling victims according to triage acuity principles. With what color tag should the nurse label a client who is experiencing respiratory distress? 1. Red 2. Black 3. Green 4. Yellow

1 A red tag (priority I) indicates a client with respiratory distress, trauma or bleeding, or neurological deficits that need immediate treatment. Victims who are deceased are labeled with a black tag. These individuals are transported to a temporary morgue after clients who have a chance for survival are attended. A green tag (priority III) indicates a client who needs care that can wait for hours. Although clients with sprains, rashes, and minor pain can wait hours for treatment, they need to be reassessed every 1 to 2 hours to ensure that their condition did not deteriorate. If their condition deteriorates they should be relabeled according to their level of need. A yellow tag (priority II) indicates injuries that need treatment within 2 hours. Although people who have sustained simple fractures, lacerations, or fevers can wait for treatment for 2 hours, they need to be reassessed every 30 minutes to ensure that their condition did not deteriorate. If their condition deteriorates they should be relabeled with a red tag (priority I) indicating the need for immediate treatment.

A client is receiving intravenous heparin for treatment for a deep vein thrombosis (DVT). While completing the client's morning assessment, the nurse notes that the intravenous (IV) insertion site is cool to touch and the skin appears blanched. The nurse also notes there is <1 inch edema around the site. Based on the infiltration scale, how should the nurse properly document the infiltration? 1. One (1) 2. Two (2) 3. Three (3) 4. Four (4)

1 According to the infiltration scale, grade one presents as skin blanched, edema <1 inch in any direction, cool to touch, and with or without pain. Grade two presents as skin blanched, edema 1--6 inches in any direction, cool to touch, and with or without pain. Grade three presents as skin blanched, translucent, edema >6 inches in any direction, cool to touch, mild to moderate pain, and possible numbness. Grade four presents as skin blanched, translucent, skin tight, leaking, skin discolored, bruised, swollen, gross edema >6 inches in any direction, deep pitting tissue edema, circulatory impairment, moderate to severe pain, and infiltration of any amount of blood product, irritant, or vesicant.

The student nurse demonstrates correct understanding of anemia related to chronic disease with which statement? 1. "Red blood cells appear normal in size and color; however, there is a decreased amount produced." 2. "The red blood cells have an increased life span with a decrease in normal functioning." 3. "Administration of vitamins B12 and folate will help to treat this type of long-term anemia." 4. "This is the mildest form of anemia and is easily corrected through administration of blood products."

1 Anemia of chronic disease results in a decrease in the production of red blood cells (RBCs) in response to chronic inflammation; the red blood cells are normal size, shape, and color. There is a decrease in the lifespan of the RBC and the administration of folate or B12 will not correct the anemia as these levels are generally within normal limits. This form of anemia can be very severe and treatment is directed at identification and management of the underlying cause.

A client has chronic asthma. For which complication should the nurse monitor this client? 1. Atelectasis 2. Pneumothorax 3. Pulmonary edema 4. Respiratory alkalosis

1 As a result of narrowed airways, adequate ventilation of lung tissue is compromised and alveoli may collapse. Pneumothorax is not a common complication of asthma; a collapsed lung is referred to as a pneumothorax. Pulmonary edema is not a common complication of asthma; pulmonary edema is caused by left-sided heart failure. Respiratory alkalosis is not a common complication of asthma; with narrowed air passages, the client with asthma is at risk for hypoxia and respiratory acidosis.

When auscultating a client's heart, the nurse understands that the first heart sound is produced by the closure of the: 1. Mitral and tricuspid valves 2. Aortic and tricuspid valves 3. Mitral and pulmonic valves 4. Aortic and pulmonic valves

1 Closure of the atrioventricular valves, the mitral and tricuspid, produces the first heart sound (S1). Aortic and tricuspid valves and mitral and pulmonic valves do not close simultaneously. Aortic and pulmonic valves are the semilunar valves; closure of these valves produces the "dub" (S2) sound.

When assessing an 85-year-old client's vital signs, the nurse anticipates a number of changes in cardiac output that result from the aging process. The finding that is consistent with a pathologic condition rather than the aging process is: 1. A pulse rate irregularity 2. Equal apical and radial pulse rates 3. A pulse rate of 60 beats per minute 4. An apical rate obtainable at the fifth intercostal space and midclavicular line

1 Dysrhythmias are abnormal and are associated with acute or chronic pathological conditions. An equal apical and radial pulse is expected; the radial pulse reflects ventricular contractions. The expected range in adults is 60 to 100 beats per minute. An apical rate obtainable at the fifth intercostal space and midclavicular line are the anatomical landmarks for locating the apex of the heart; they are unaffected by aging.

A primary health care provider prescribes a heart-healthy diet for a client with angina. The client's spouse says to the nurse, "I guess I'm going to have to cook two meals, one for my spouse and one for myself." Which is the most appropriate response by the nurse? 1. "The diet prescribed for your spouse is a healthy diet. It contains guidelines that many of us should follow." 2. "I wouldn't bother. For this diet all that you need to do is to reduce the amount of salt you use and fry foods in peanut oil." 3. "You're right. Be careful to cook a small proportion for each of you to eat to not waste food." 4. "This is a difficult diet to follow. I recommend that you shop daily for food so there are no temptations in the kitchen."

1 Heart-healthy diets are low in cholesterol, sodium, and fat, particularly saturated fats, and high in vegetables and fruits; this type of diet is advocated for all individuals. Fried foods are not advocated on a heart-healthy diet; peanut oil is a monounsaturated fatty acid, and these acids should not exceed 15% of the calories of the diet. The responses "You're right. Be careful to cook a small proportion for each of you to eat to not waste food" and "This is a difficult diet to follow. I recommend that you shop daily for food so there are no temptations in the kitchen" can be discouraging and encourages noncompliance.

A client is admitted with multiple injuries as a result of an accident. A tracheostomy was performed. While the nurse is caring for this client, the client coughs, expelling the tracheostomy tube onto the bed. What should the nurse's first action be? 1. Hold the tracheostomy open with a tracheal dilator and call for assistance 2. Insert an obturator into the tracheostomy and gently reinsert the tracheostomy tube 3. Pick up the tracheostomy tube from the bed and replace it until a new tube is available 4. Obtain a new tracheostomy tube, prepare the new holder, and insert the tube using the obturator

1 Holding the tracheostomy open with a tracheal dilator and calling for assistance provides an immediate airway without causing trauma; with assistance, a new tracheostomy tube can be inserted. The obturator will obstruct the airway. Replacing the tube that fell on the bed linen is contraindicated because it is contaminated; a sterile tube should be inserted. If the airway is not held open, the client will experience hypoxia.

A client has a femoral-popliteal bypass graft. When a nurse assesses the vital signs, the client's blood pressure is 200/110 mm Hg. Why should the nurse notify the health care provider immediately? 1. Graft may rupture. 2. Client is anaphylactic. 3. Client is hypervolemic. 4. Graft may be occluded.

1 Hypertension increases pressure on the suture lines of the graft. Signs of shock, including a drop in blood pressure, are associated with a severe allergic reaction. An increased blood pressure does not necessarily mean that the client is hypervolemic. An occluded graft is indicated by absent pedal pulses.

What change in pressure does the nurse conclude is responsible for the lower extremity pitting edema of a client with right ventricular heart failure? 1. Increase in plasma hydrostatic pressure 2. Increase in tissue colloid osmotic pressure 3. Decrease in the tissue hydrostatic pressure 4. Decrease in the plasma colloid osmotic pressure

1 In right ventricular heart failure, blood backs up in the systemic capillary beds; the increase in plasma hydrostatic pressure shifts fluid from the intravascular compartment to the interstitial spaces, causing edema. Increase in tissue colloid osmotic pressure occurs with crushing injuries or if proteins pathologically shift from the intravascular compartment to the interstitial spaces. Increased fluid pressure in the intravascular compartment causes fluid to shift to the tissues; the tissue hydrostatic pressure does not decrease. Although a decrease in colloid osmotic (oncotic) pressure can cause edema, it results from lack of protein intake, not increased hydrostatic pressure associated with right ventricular heart failure. Test-Taking Tip: After choosing an answer, go back and reread the question stem along with your chosen answer. Does it fit correctly? The choice that grammatically fits the stem and contains the correct information is the best choice.

After emergency surgery, the nurse teaches a client how to use an incentive spirometer. What client behavior indicates to the nurse that the spirometer is being used correctly? 1. Inhales deeply through the mouthpiece, relaxes, and then exhales. Incorrect2 Inhales deeply, seals the lips around the mouthpiece, and exhales. 3 Uses the incentive spirometer for 10 consecutive breaths per hour. 4 Coughs several times before inhaling deeply through the mouthpiece.

1 Inhaling deeply through the mouthpiece, relaxing, and then exhaling are correct techniques; deep inhalation promotes alveolar expansion, and exhalation promotes lung recoil. Inhaling deeply, sealing the lips around the mouthpiece, and exhaling are incorrect techniques; inhalation should occur through the mouthpiece. The breaths should not be taken in succession; they should be spaced by several normal breaths to avoid fatigue. Coughing is done after deep breathing.

Within the first 2½ hours after a radical neck dissection, 40 mL of medium red, bloody fluid is collected in the portable wound drainage system. What should the nurse do first? 1. Obtain the vital signs. 2. Notify the health care provider. 3. Chart the amount on the intake & output (I&O) record. 4. Continue to monitor the amount for another 2½ hours.

1 Obtaining the vital signs shows an excessive amount of drainage in 2½ hours because 80 to 120 mL of drainage is expected within the first 24 postoperative hours. The client's status should be assessed first, then the health care provider should be notified. Charting the amount on the I&O record should be done eventually, but it is not the priority at this time. Continuing to monitor the amount for another 2½ hours is unsafe; the client is bleeding excessively. Test-Taking Tip: Be alert for details. Details provided in the stem of the item, such as behavioral changes or clinical changes (or both) within a certain time period, can provide a clue to the most appropriate response or, in some cases, responses.

Oxygen given to clients during stage 4 of chronic obstructive pulmonary disease (COPD) should be administered in which manner? 1. 1 to 2 L via nasal cannula to keep SaO2 above 90%. 2 1 to 2 L via nasal cannula to maintain SaO2 at or above 95%. 3 3 L via mask to maintain SaO2 at 95%. 4 Do not give oxygen because it may suppress hypoxic drive in client.

1 Oxygen therapy usually is delayed until stage 4, which is very severe COPD. Usually it is administered at 1 to 2 L per minute to maintain SaO2 at or above 90%. One to 2 L to maintain the SaO2 above 95% is not necessary. Oxygen administration may not be necessary. Three liters of oxygen via a mask is unnecessary and a level of 95% may suppress the hypoxic drive in clients who are chronic CO2 retainers. Oxygen should not be given unless the chronic saturation level is less than 88%.

A client is receiving total parenteral nutrition (TPN) through a central venous access device. Important nursing interventions include: 1. Placing the client in the supine position before changing the tubing 2 Monitoring the blood pressure frequently to assess for hypovolemia 3 Decreasing the infusion rate if blood glucose levels become elevated 4 Piggybacking intravenous antibiotics onto the TPN tubing to prevent infection

1 Placing the client in the supine position before changing the tubing decreases pressure in the vena cava, which helps prevent an air embolus when the catheter is disconnected. Infusion of high concentrations of glucose will cause hypervolemia, not hypovolemia. The infusion rate is changed only with a health care provider's prescription. Although insulin is contained in the parenteral nutrition formula, when blood glucose levels become elevated the health care provider may prescribe insulin coverage. No medications or solutions other than the parenteral nutrition should be administered through this line.

A client with laryngeal cancer has a partial laryngectomy and tracheostomy. To best facilitate communication postoperatively, the nurse should: 1. Provide a means for the client to write. 2 Allow the client more time for articulation. 3 Use visual clues, such as gestures and objects. 4 Face the client and speak slowly and distinctly.

1 The client will be unable to speak because a tracheostomy tube is in place to prevent edema. The client cannot speak with a tracheostomy tube in place. The client's ability to see and hear is not affected. The client can receive information but cannot speak. The client's ability to hear or understand is not affected.

A 28-year-old woman fractured her left tibia and fibula one week ago and has a cast in place. She is taking acetaminophen (Tylenol) with codeine for pain and an oral contraceptive. She began experiencing left calf pain 3 days ago and began having shortness of breath and chest pain 15 minutes ago. When the shortness of breath and chest pain increase, she calls the emergency department and communicates this information to the triage nurse. What is the nurse's best response? 1. "Give me your name and address. I am sending an ambulance to your home. You need emergency care." 2. "It sounds as if your cast may be constricting the blood flow in your leg. You probably need a new cast." 3. "It sounds like you are having an allergic response to the medication. Can you drive yourself to the hospital?" 4. "You are experiencing a pulmonary embolism. You need to come to the emergency department now for care."

1 The client's clinical manifestations, along with the history of a recent fracture, immobilization, and use of an oral contraceptive, suggest a pulmonary embolism. An ambulance will limit the woman's use of her leg, which may prevent further emboli. The client's clinical findings are not indicative of compression syndrome. Tingling, numbness, cool skin, and lack of capillary refill are signs and symptoms of compression syndrome. Activity will increase shortness of breath and may precipitate the release of additional emboli. Although the client may be experiencing a pulmonary embolism, making a medical diagnosis is outside the nurse's responsibilities and may increase the client's anxiety.

During admission a client appears anxious and says to the nurse, "The doctor told me I have lung cancer. My father died from cancer. I wish I had never smoked." What is the nurse's best response? 1. "You are concerned about your diagnosis." 2. "You are feeling guilty about your smoking." 3. "There have been advances in lung cancer therapy." 4. "Trust your health care provider, who is very competent in treating cancer."

1 The correct response recognizes and acknowledges the client's concerns without assuming a specific feeling is involved; it allows the client to set the framework for discussion and express self-identified feelings. The client's statement is not specific enough to come to the conclusion that the client feels guilty; this is an assumption by the nurse. Talking about advances in lung cancer therapy or trust for the health care provider avoid the client's concerns and cut off communication.

A client is on a cardiac monitor. The monitor begins to alarm showing ventricular tachycardia. What should the nurse do first? 1. Check for a pulse. 2. Start cardiac compressions. 3. Administer oxygen via an ambu bag. 4. Prepare to defibrillate the client.

1 The treatment of ventricular tachycardia depends on the presence of a pulse. Therefore, checking for a pulse is the first priority for the nurse. Cardiac compressions would not be initiated if there was a pulse. Administering oxygen via an ambu bag would only occur if the client was not breathing. The client is not automatically defibrillated. Cardioversion is recommended for slower ventricular tachycardia.

What potentially adverse effect of an intravenous titrated drip of lidocaine (Xylocaine) should the nurse immediately report to the healthcare provider? 1. Tremors 2 Anorexia 3 Tachycardia 4 Hypertension

1 Tremors are a precursor to the major adverse effect of seizures. Although anorexia may occur, it is not a serious side effect. Bradycardia, which may lead to heart block, may occur, not tachycardia. Hypotension, not hypertension, may occur.

Serum cardiac marker studies are prescribed for a client after a myocardial infarction. Which laboratory test is most important for the nurse to monitor because it has the greatest sensitivity and specificity for myocardial injury? 1. Troponin 2. Myoglobin 3. Creatine kinase (CK) 4. Lactate dehydrogenase (LDH)

1 Troponin, specifically subtypes cardiac-specific troponin T (cTnT) and cardiac-specific troponin I (cTnI), reflects myocardial muscle protein released into circulation soon after injury. Troponin increases as quickly as CK and remains increased for 2 weeks. Although myoglobin is one of the first cardiac markers that increase after a myocardial infarction (MI), it lacks cardiac specificity. CK isoenzyme levels, especially the creatine phosphokinase (MB) subunit, begin to increase in 3 to 6 hours, peak in 12 to 18 hours, and are increased for 48 hours after the occurrence of the infarct. Although reliable in assisting with an early diagnosis of MI, it is not as sensitive or specific as the troponin test. LDH isoenzyme levels, especially LDH-1, do not begin to increase until 12 hours after the infarct and peak in 48 hours, so they are a much later indicator and therefore are not as significant.

A client is receiving warfarin (Coumadin). Which test result should the nurse use to determine if the daily dose of this anticoagulant is therapeutic? 1. International normalized ratio (INR) 2 Accelerated partial thromboplastin time (APTT) 3 Bleeding time 4 Sedimentation rate

1 Warfarin initially is prescribed day by day, based on INR blood test results. This test provides a standard system to interpret prothrombin times. APTT is used to evaluate the effects of heparin, which acts on the intrinsic pathway. Bleeding time is the time required for blood to cease flowing from a small wound; it is not used for warfarin dosage calculation. Sedimentation rate is a test used to determine the presence of inflammation or infection; it does not indicate clotting ability.

A client with cancer of the lung says to the nurse, "If I could just be free of pain for a few days, I might be able to eat more and regain strength." Which stage of grieving does the nurse conclude the client is in? 1. Bargaining 2 Frustration 3 Depression 4 Rationalization

1 Bargaining is one of the stages of grieving, in which the client promises some type of desirable behavior to postpone the inevitability of death. Frustration is a subjective experience, a feeling of being thwarted, but it is not one of the stages of grieving. Classified as the fourth stage of grieving, depression represents the grief experienced as the individual recognizes the inescapability of fate. Rationalization is a defense mechanism in which attempts are made to justify or explain an unacceptable action or feeling; it is not a stage of the grieving process.

A nurse is caring for a client who had a pneumonectomy. What is the priority nursing assessment? 1. Pulse oximetry 2. Ventilatory exchange 3 Closed-chest drainage 4 Approximation of the incision

2 Gas exchange is of primary importance to promote life. Although pulse oximetry should be monitored, it is not the priority. Although the closed-chest drainage system should be monitored, it is not the priority. Although approximation of the incision should be monitored, it is not the priority.

A health care provider prescribes doxorubicin for a client with acute myelogenous leukemia. Which specific interventions should the nurse implement? Select all that apply. 1. Monitor for jaundice 2 Increase fluids by mouth 3 Provide frequent oral care 4 Increase physical activities 5 Assess vital signs routinely 6 Serve hot liquids with meals

1, 2, 3, 5 Doxorubicin is hepatotoxic; the client should be assessed for jaundice. Hyperuricemia is a possible complication of therapy; therefore, adequate hydration is important. Stomatitis is a possible complication of therapy; therefore, oral care is important to help maintain the integrity of the oral mucous membranes. Hypertension, sinus tachycardia, bradycardia, premature ventricular complexes, and asystole may occur with therapy. Leukopenia may make the client vulnerable to infection, with an associated increase in temperature. Anemia is a common side effect of therapy; rest is important because of the increased fatigability associated with therapy. Extremes in temperature should be avoided when administering fluids and food because of the common occurrence of stomatitis.

After surgery, a client is extubated in the postanesthesia care unit. Which clinical manifestations should the nurse expect if the client is experiencing acute respiratory distress? Select all that apply. 1. Confusion 2. Hypocapnia 3. Tachycardia 4. Constricted pupils 5. Slow respiratory rate

1, 3 Inadequate cerebral oxygenation produces restlessness and confusion. Tachycardia occurs as the body attempts to compensate for the lack of oxygen. A low carbon dioxide level in the blood (hypocapnia) occurs with hyperventilation, not hypoxia. The pupils dilate, not constrict, with hypoxia. An elevated respiratory rate (tachypnea), not a slow respiratory rate (bradypnea), occurs.

For which side effects should a nurse assess a client with cancer who is being treated with chemotherapeutic agents? Select all that apply. 1. Diarrhea 2. Leukocytosis 3. Bleeding tendencies 4. Lowered sedimentation rate 5. Increased hemoglobin levels

1, 3 Most chemotherapeutic agents interfere with mitosis. The rapidly dividing cells of the mucous membranes of the gastrointestinal tract are affected, causing stomatitis and diarrhea. Bone marrow depression often causes thrombocytopenia, resulting in bleeding tendencies. Most chemotherapeutic agents interfere with mitosis. The bone marrow consists of rapidly dividing cells, and therefore its activity is depressed. Leukopenia, not leukocytosis, can occur. The erythrocyte sedimentation rate (ESR) generally increases in the presence of tissue inflammation or necrosis. Hemoglobin and hematocrit levels may decrease because of an inadequate number of red blood cells related to bone marrow depression.

A beta-blocker, atenolol (Tenormin), is prescribed for a client with moderate hypertension. What information should the nurse include when teaching the client about this medication? Select all that apply. 1. Change positions slowly 2. Take the medication before going to bed 3. Count the pulse before taking the medication 4. Mild weakness and fatigue are common side effects 5. It is safe to take concurrent over-the-counter (OTC) medications

1, 3, 4 A side effect of this medication is orthostatic hypotension. The client should be advised to move to a standing position slowly to allow the vasomotor response of the body to adjust to the new position. The rate of the pulse should be taken before administering the medication; ventricular dysrhythmias and heart block may occur. Mild weakness and fatigue, as well as dizziness and depression, are side effects of this medication. The blood pressure decreases when the client is sleeping; the medication usually is prescribed to be administered earlier in the day. The medication should be taken with food. No OTC medication should be taken without consulting the prescribing health care provider; decreased or increased effects can occur when there is an interaction with another medication.

A health care provider's prescription for a client in ketoacidosis is 500 units Humulin R insulin in 500 mL lactated Ringers intravenously (IV), and administer 60 units of insulin in the next 30 minutes. At how many milliliters per hour should the nurse set the IV infusion device to administer the correct amount of medication? Record your answer using a whole number. ____ mL/hr

120 120 mL/hr is the correct flow rate. The rate of solution to be administered on an IV infusion device is set at mL/hr; 500 units of insulin added to 500 mL of IV fluid results in a solution in which 1 mL contains 1 unit of the drug; therefore, 60 mL = 60 units to be administered over 30 minutes. Solve for x using ratio and proportion.

A client experiences crushing chest pain and is brought to the emergency department. When assessing the ECG tracing, the nurse concludes that the client is experiencing premature ventricular complexes (PVCs). Which abnormalities of the electrocardiogram support this conclusion? 1. Irregular rhythm, abnormal shaped P wave, and normal QRS 2 Irregular rhythm, absence of a P wave, wide, and distorted QRS 3 Regular rhythm, more than 100 beats per minute, normal P wave, and normal QRS 4 Regular rhythm, 100 to 250 beats per minute, absent P wave, and wide, distorted QRS

2 A PVC is a contraction originating in an ectopic focus in the ventricles; it is characterized by a premature, wide, distorted QRS complex with the P wave and PR interval buried in the distorted QRS complex, resulting in an irregular rhythm. Irregular rhythm, abnormal shaped P wave, and normal QRS occur with a premature atrial complex. Regular rhythm, more than 100 beats per minute, normal P wave, and normal QRS occur with sinus tachycardia. Regular rhythm, 100 to 250 beats per minute, absent P wave, and wide, distorted QRS occur with ventricular tachycardia.

The nurse is analyzing the client's rhythm when she notes multiple premature ventricular contractions (PVCs). Each PVC occurs in no particular pattern and looks like all other PVCs. This indicates that the PVCs are: 1. Multifocal. 2. Unifocal. 3. Bigeminal. 4. A pair.

2 A single ectopic focus produces PVC waveforms that look alike, called unifocal PVCs. Waveforms of PVCs arising from multiple foci are not identical and are called multifocal PVCs. PVCs may occur in a predictable pattern, such as every other beat (bigeminal), every third beat (trigeminal), or every fourth beat (quadrageminy). PVCs also can occur sequentially. Two PVCs in a row are called a pair, and three or more in a row are called nonsustained ventricular tachycardia.

A client is receiving warfarin (Coumadin) for a pulmonary embolism. Which drug is contraindicated when taking warfarin? 1. Ferrous sulfate 2. Acetylsalicylic acid (aspirin) 3. Atenolol (Tenormin) 4. Chlorpromazine (Thorazine)

2 Acetylsalicylic acid can cause decreased platelet aggregation, increasing the risk for undesired bleeding that may occur with administration of anticoagulants. Ferrous sulfate does not affect warfarin; it is used for red blood cell synthesis. Atenolol is a beta blocker that reduces blood pressure; it does not affect bleeding. Chlorpromazine is a neuroleptic; it does not affect bleeding.

A nurse is caring for a client who is a victim of trauma and is to receive a blood transfusion. How should the nurse respond when the client expresses fear that acquired immunodeficiency syndrome (AIDS) may be acquired as a result of the blood transfusion? 1. "The blood is treated with radiation to kill the virus." 2. "Screening for the human immunodeficiency virus (HIV) antibodies has minimized this risk." 3. "The ability to directly identify HIV has eliminated this concern." 4. "Consideration should be given to donating your own blood for transfusion."

2 Although blood is screened for the antibodies, there is a period between the time a potential donor is infected and the time when antibodies are detectable; there is still a risk, but it is minimal. There is no current method of destroying the virus in a blood transfusion. The screening tests involve identification of the antibody, not the virus itself; the virus can be identified by the polymerase chain reaction test but is not part of routine screening. Although many people consider autotransfusion for elective procedures, a trauma victim does not have this option.

For what client response must the nurse monitor to determine the effectiveness of amiodarone (Cordarone)? 1. Results of fasting lipid profile 2. Presence of cardiac dysrhythmias 3. Degree of blood pressure control 4. Incidence of ischemic chest pain

2 Amiodarone is a class III antidysrhythmic used to treat ventricular and supraventricular tachycardia, and conversion of atrial fibrillation. Results of fasting lipid profile are expected with antilipidemics. Degree of blood pressure control is expected with antihypertensives. Incidence of ischemic chest pain is expected with antianginal agents, such as nitrates.

A client is to continue oxygen therapy at home when discharged. Which client statement indicates the need for further instruction by the nurse? 1. "I will use only grounded electrical equipment." 2 "I have a new woolen blanket to keep me warm." 3 "I have told my family they cannot smoke in the house." 4 "I will keep a pitcher of water near me so I drink enough."

2 An open flame or a spark from static electricity (generated by such items as leather-soled shoes; wool, silk, nylon, and Dacron blankets; or ungrounded electrical appliances) can initiate an explosion and fire in the presence of higher-than-environmental oxygen levels. Grounded electrical equipment helps prevent sparks. When combined with oxygen, heat from lit cigarettes can ignite flammable material. Oxygen is drying; increased fluid intake is advisable.

During a blood transfusion a client develops chills and a headache. What is the priority nursing action? 1. Cover the client. 2. Stop the transfusion at once. 3. Decrease the rate of the blood infusion. 4. Notify the health care provider immediately.

2 Chills, headache, nausea, and vomiting are all signs of a transfusion reaction. The infusion must be stopped before treatment of symptoms begins. Slowing the infusion will continue the reaction, which may lead to kidney damage. The health care provider should be notified after the transfusion is stopped. Study Tip: Record the information you find to be most difficult to remember on 3" × 5" cards and carry them with you in your pocket or purse. When you are waiting in traffic or for an appointment, just pull out the cards and review again. This "found" time may add points to your test scores that you have lost in the past.

A nurse is teaching a group of clients with peripheral vascular disease about a smoking cessation program. Which physiologic effect of nicotine should the nurse explain to the group? 1. Constriction of the superficial vessels dilates the deep vessels. 2. Constriction of the peripheral vessels increases the force of flow. 3. Dilation of the superficial vessels causes constriction of collateral circulation. 4. Dilation of the peripheral vessels causes reflex constriction of visceral vessels.

2 Constriction of the peripheral blood vessels and the resulting increase in blood pressure impair circulation and limit the amount of oxygen being delivered to body cells, particularly in the extremities. Nicotine constricts all peripheral vessels, not just superficial ones. Its primary action is vasoconstriction; it will not dilate deep vessels. Nicotine constricts rather than dilates peripheral vessels.

A client is in profound (late) hypovolemic shock. The nurse assesses the client's laboratory values. Clients that are in late shock develop: 1. Hypokalemia 2. Metabolic acidosis 3. Respiratory alkalosis 4. Decreased PCO2 levels

2 Decreased oxygen increases the conversion of pyruvic acid to lactic acid, resulting in metabolic acidosis. Hyperkalemia will occur because of renal shutdown; hypokalemia can occur in early shock. Respiratory alkalosis can occur in early shock because of rapid, shallow breathing, but in late shock metabolic or respiratory acidosis occurs. The PCO2 level will increase in profound shock. Study Tip: Remember that intelligence plays a vital role in your ability to learn. However, being smart involves more than just intelligence. Being practical and applying common sense are also part of the learning experience.

A nurse is caring for a client with a history of hypertension who was admitted to the hospital with aphasia. A bruit is heard over the left carotid artery, and the pulse is irregular. What is a reason for the aphasia that the nurse should consider? 1. Chronic hypertensive disease 2. Emboli associated with atrial fibrillation 3. Developmental defects of the arterial wall 4. Inappropriate paroxysmal neural discharge

2 Emboli, which may occur from atrial fibrillation, cause complete occlusion of vessels; usually, middle cerebral arteries are involved. The infarct may cause hemiplegia, aphasia, or spatial perceptual deficits. Hypertension is a disease that may cause spasm of the arteries but does not cause anatomic occlusion. Developmental defects in the arterial wall are associated with saccular aneurysms. Seizures, not aphasia, are caused by inappropriate paroxysmal discharge.

A blood transfusion is initiated after a client had emergency surgery. What should the nurse do first when the client develops fever, chills, and low back pain? 1. Notify a health care provider. 2. Stop the blood and infuse saline. 3. Obtain a prescription for an antihistamine. 4. Slow the rate of the transfusion and inform the blood bank.

2 Fever, chills, and low back pain indicate an acute hemolytic reaction, which is potentially life threatening; discontinuing the transfusion immediately limits kidney damage. The vein is kept open by running the primary bottle of normal saline. Notifying the health care provider can be done later; the client's safety must be addressed first. Obtaining a prescription for an antihistamine may be done later. Although the blood bank generally is notified if a reaction occurs, slowing the transfusion rate is unsafe because the reaction will continue.

An older client who lives alone was found unconscious on the floor at home. The client was admitted to the hospital with the diagnoses of a fractured hip, kidney failure, and dehydration. In the 24 hours since admission, the client received 1500 mL of intravenous fluid and the serum electrolyte value demonstrates hyponatremia. The nurse concludes that the element that most likely contributed to the hyponatremia is: 1. Salt intake 2. Fluid intake 3. Sodium absorption 4. Glomerular filtration

2 Hemodilution has occurred because the 1500 mL of intravenous fluid has lowered the serum sodium level. An increase in salt intake is not the cause of the hyponatremia; in addition, the client has not eaten for several days. A decreased, not increased, reabsorption of sodium occurs in acute renal failure. A decreased, not increased, glomerular filtration rate occurs with renal failure.

A client is diagnosed with heart failure and is admitted for medical management. Which statement made by the client may indicate worsening heart failure? "I am unable to run a mile now." 2 "I wake up at night short of breath." 3 "My shoes seem larger lately." 4 "My wife says I snore very loudly."

2 Increased shortness of breath is often an indicator of fluid overload in the heart failure client. Being unable to run a mile, snoring loudly, and shoes seeming larger are not related to heart failure. Study Tip: In the first pass through the exam, answer what you know and skip what you do not know. Answering the questions you are sure of increases your confidence and saves time. This is buying you time to devote to the questions with which you have more difficulty.

What is the most important information the nurse and the rapid response team must keep in mind when caring for a client who had a cardiac arrest? 1. Age of the client Correct2 How long the client was anoxic 3 Heart rate of the client before the arrest 4 Emergency medications available for the client

2 Irreversible brain damage will occur if a client is anoxic for more than four minutes. The age of the client does not affect the response by the arrest team. The earlier heart rate is of minimal importance; the rhythm is more significant. Although a variety of emergency medications must be available, their administration is prescribed by the health care provider.

A client with cancer is receiving a multiple-chemotherapy protocol. Included in the protocol is leucovorin. The nurse concludes that this drug was prescribed because leucovorin: 1. Potentiates effects of alkylating agents 2. Diminishes toxicity of folic acid antagonists 3. Limits nausea and vomiting associated with chemotherapy 4. Interferes with cell division at a different stage than other drugs

2 Leucovorin limits the toxicity of folic acid antagonists, such as methotrexate sodium, by competing for transport into the cells. Potentiating effects of alkylating agents is not the action of leucovorin. Limiting nausea and vomiting associated with chemotherapy is the purpose of antiemetics. Leucovorin calcium does not interfere with cell division. Interfering with cell division at different stages is the purpose of a variety of drugs in a multiple-drug protocol.

Which statement by the unlicensed assistive personnel (UAP) indicates a correct understanding of the UAP's role? "I will: 1. Turn off clients' IVs that have infiltrated." 2. Take clients' vital signs after their procedures are over." 3. Use unit written materials to teach clients before surgery." 4. Help by giving medications to clients who are slow in taking pills."

2 Monitoring vital signs after procedures is within the scope of a UAP's role. Registered professional nurses or licensed practical nurses, not UAPs, should perform turning off clients' IVs that have infiltrated. Using unit written materials to teach clients before surgery should be performed by registered professional nurses or licensed practical nurses, not UAPs. Helping by giving medications to clients who are slow in taking pills should be performed by registered professional nurses or licensed practical nurses, not UAPs.

A client is diagnosed with pancytopenia caused by chemotherapy. What should a nurse teach the client about this complication? 1. Begin a program of meticulous mouth care. 2 Avoid traumatic injury and exposure to infection. 3 Increase oral fluid intake to at least 3 L a day. 4 Report unusual muscle cramps or tingling sensations in the extremities.

2 Reduced platelets increase the likelihood of uncontrolled bleeding; reduced lymphocytes increase the susceptibility to infection. Beginning a program of meticulous mouth care is helpful for stomatitis, not pancytopenia; aggressive oral hygiene may precipitate bleeding from the gums. Although fluids may be increased to flush out the toxic byproducts of chemotherapy, this will have no effect on pancytopenia. Unusual muscle cramps or tingling sensations in the extremities are signs of hypocalcemia and do not apply to pancytopenia.

A nurse is caring for a client with a pneumothorax that has a chest tube attached to a closed chest drainage system. If the chest tube and closed-chest drainage system are effective, the type of pressure that will be reestablished is: 1. Neutral pressure in the pleural space 2. Negative pressure in the pleural space 3. Atmospheric pressure in the thoracic cavity 4. Intrapulmonic pressure in the thoracic cavity

2 Removal of air and fluid from the pleural space reestablishes negative pressure, resulting in lung expansion. Neutral pressure in the pleural space will cause collapse of the lung. Atmospheric pressure in the thoracic cavity will cause collapse of the lung. Intrapulmonic pressure refers to pressure within the lung itself, not the pressure within the thoracic cavity.

Two portable drainage catheters with hemovacs attached were placed during a client's hemiglossectomy and right radical neck dissection. Six hours after the catheters were placed, the nurse empties 180 mL of serosanguineous drainage from one of the drainage catheters. The priority nursing intervention is to: 1. Turn the client onto the right side 2. Notify the health care provider immediately 3. Document the output as an expected finding 4. Irrigate the drainage catheter to ensure patency

2 Serosanguineous drainage of 80 to 120 mL is expected during the first 24 hours; more than this amount of drainage should be reported. Placing the client in the side-lying position will have no effect on the portable wound drainage system; it functions via negative pressure, not gravity. Drainage of 180 mL in six hours is excessive and should be reported. It is unusual for drainage catheters to need irrigation to remain patent. It is evident that the catheter is not obstructed.

During auscultation of the heart, the nurse expects the first heart sound (S1) to be the loudest at the: 1. Base of the heart 2. Apex of the heart 3. Left lateral border 4. Right lateral border

2 The first heart sound is produced by closure of the mitral and tricuspid valves; it is heard best at the apex of the heart. Base of the heart is where the second heart sound (S2) is best heard; S2 is produced by closure of the aortic and pulmonic valves. Left lateral border covers a large area; the auscultatory areas that lie near it are the pulmonic and mitral areas. Right lateral border covers a large area; the only auscultatory area near it is the aortic area.

A client has arrived on the nursing unit scheduled for a cholecystectomy, and there is one registered nurse, a licensed practical nurse, and a certified nursing assistant. Institution policy is that all admissions must have the initial assessment completed by a registered nurse. The registered nurse currently is caring for a client with a transfusion reaction. Which is the best decision? 1. Tell the practical nurse to admit the patient and complete documentation. 2. Have the practical nurse perform the initial assessment and the registered nurse verify all findings after the transfusion client is stable. 3. Instruct the certified nursing assistant to obtain vital signs every 15 minutes as this is within the scope of practice. 4. Tell the admission clerk to put the client in the designated room and explain that the nurses are busy with an emergency and will get to the client as soon as possible.

2 The practical nurse is able to perform the initial assessment but the registered nurse must verify and sign for all findings. The transfusion client is a priority until stable and warrants care by the registered nurse. Institution policy dictating initial assessment by the registered nurse will be met by this plan. Telling the practical nurse to do the assessment and completing documentation does not include verification by the registered nurse, which is in the institution policy. The certified nursing assistant is capable of taking vital signs but does not have the assessment skills necessary to care for an acute client. Instructing the admission personnel to place a client in a room and stating that the nurses are too busy would not inspire confidence.

A client is diagnosed with hypertension that is related to atherosclerosis. The nurse recalls that with atherosclerosis: 1. Rennin causes a gradual decrease in arterial pressure 2. Lipid plaque formation occurs within the arterial vessels 3 Mobilization of free fatty acid from adipose tissue contributes to plaque formation 4 Development of atheromas within the myocardium is characteristic

2 The term atherosclerosis means a thickening of the arterial lining by lipid plaques, which become atheromas. Arterial pressure increases, not decreases, as a result of rennin. Mobilization of free fatty acids will produce an acid-base imbalance. Atheromas develop within the lining of the arteries, not within the cardiac muscle tissue.

A client who has acquired human immunodeficiency syndrome (HIV) develops bacterial pneumonia. On admission to the emergency department, the client's PaO2 is 80 mm Hg. When the arterial blood gases are drawn again, the level is determined to be 65 mm Hg. What should the nurse do first? 1. Prepare to intubate the client. 2 Increase the oxygen flow rate per facility protocol. 3 Decrease the tension of oxygen in the plasma. 4 Have the arterial blood gases redone to verify accuracy.

2 This decrease in PaO2 indicates respiratory failure; it warrants immediate medical evaluation. Most facilities have a protocol to increase the oxygen flow rate to keep oxygen saturation greater than 92%. The client PaO2 of 65 mm Hg is not severe enough to intubate the patient without first increasing flow rate to determine if the client improves. Decreasing the tension of oxygen in the plasma is inappropriate and will compound the problem. The PaO2 is a measure of the pressure (tension) of oxygen in the plasma; this level is decreased in individuals who have perfusion difficulties, such as those with pneumonia. Having the arterial blood gases redone to verify accuracy is negligent and dangerous; a falling PaO2 level is a serious indication of worsening pulmonary status and must be addressed immediately. Drawing another blood sample and waiting for results will take too long.

A client who had a myocardial infarction receives 15 mg of morphine sulfate for chest pain. Fifteen minutes after receiving the drug, the client complains of feeling dizzy. What action should the nurse take? 1. Determine if this is an allergic reaction 2. Place the client in the supine position and take the vital signs 3. Elevate the client's head and keep the extremities warm 4. Tell the client that this is not a typical sensation after receiving morphine sulfate

2 Vertigo is a symptom of hypotension, a side effect of morphine sulfate. The supine position increases venous return, increases cardiac output, and increases blood flow to the brain. Dizziness is a symptom of hypotension that is a side effect, not an allergic response, to morphine sulfate. Raising the client's head may aggravate dizziness. Dizziness is a typical side effect of morphine sulfate.

A nurse is caring for a client who is receiving a unit of packed red blood cells. Which findings lead the nurse to suspect a transfusion reaction caused by incompatible blood? Select all that apply. 1. Cyanosis 2 Backache 3 Shivering 4 Bradycardia 5 Hypertension

2, 3 Mismatched blood cells are attacked by antibodies, and the hemoglobin released from ruptured erythrocytes plugs the kidney tubules; this kidney involvement results in backache. Shivering occurs as part of the inflammatory response associated with a transfusion reaction. Cyanosis is not commonly associated with a transfusion reaction. Tacycardia, not bradycardia, is associated with a transfusion reaction. Hypotension, not hypertension, is associated with a transfusion reaction.

The nurse is providing postoperative care for a client who has received a prescription for nalbuphine (Nubain) for pain. For which side effects or adverse reactions should the nurse assess this client after administering this medication? Select all that apply. 1. Oliguria 2 Dry mouth 3 Headache 4 Abdominal cramps 5 Urinary retention 6 Hypotension

2, 3, 4, 6 Dry mouth, headache, and abdominal cramps are side effects of nalbuphine HCl. Hypotension may occur with nalbuphine HCl. The ability to form urine is not affected; an increased urinary output or frequency may occur. Urinary urgency, not retention, is a reaction to nalbuphine HCl.

The nurse is planning to teach a client with heart failure about the signs and symptoms of cardiac decompensation. What clinical manifestations should the nurse include? Select all that apply. 1. Weight loss 2. Extreme fatigue 3. Coughing at night 4. Excessive urination 5. Difficulty breathing

2, 3, 5 Fatigue is caused by a lack of adequate oxygenation of body cells caused by a decreased cardiac output. As the cardiac output decreases, pulmonary congestion increases, resulting in pulmonary edema; coughing, especially when one is lying down, and blood-tinged sputum occur. Auscultation reveals crackles and rhonchi. Dyspnea is associated with pulmonary edema that occurs as cardiac output decreases and pulmonary congestion increases. Weight gain, not loss, occurs as fluid is retained by the kidneys. Fluid retention, not diuresis, occurs because of decreased circulation to the kidneys, resulting from decreased cardiac output.

A client has left ventricular heart failure. For which clinical indicators should the nurse assess the client? Select all that apply. 1. Ascites 2. Crackles 3. Peripheral edema 4. Dyspnea on exertion 5. Jugular vein distention

2, 4 Pressure in the pulmonic circulation increases when the left ventricle fails; fluid moves from the intravascular compartment into the alveoli, causing crackles. Pressure in the pulmonic circulation increases when the left ventricle fails; fluid in the alveoli impairs gas exchange, which causes dyspnea. Ascites, a sign of right ventricular failure, results from an increased hydrostatic pressure in the systemic circulation; fluid moves out of the intravascular compartment into the abdominal cavity. Peripheral edema, a sign of right ventricular failure, results from an increased hydrostatic pressure in the systemic circulation. Fluid moves out of the intravascular compartment into the interstitial compartment. Jugular vein distention, a sign of right ventricular failure, results from hypervolemia.

A healthcare provider prescribes a medication to be administered via a metered dose inhaler (MDI) for a young adult with asthma. List in order the steps the nurse teaches the client to follow when using the inhaler. 1. Hold the inhaler upright in the mouth 2. Shake the inhaler for 30 seconds 3. Start breathing in and press down on the inhaler once 4. Exhale slowly and deeply to empty the air from the lungs

2, 4, 1, 3 When using an MDI, the medication should be shaken for 30 seconds to ensure that the medication is mixed. Exhaling completely maximizes emptying the lungs. The inhaler should be held upright in the mouth past the teeth with the lips closed around the mouthpiece (closed mouth method) or held upright 1 to 2 cm in front of the open mouth (open mouth method). Inhalation is begun at the same time that the device is compressed to ensure that maximum medication reaches the lungs.

What statement is true of ventricular dysrhythmias? Select all that apply. 1. The stimulus depolarizes the ventricles faster than normal. 2. The QRS complex appears widened and has a bizarre shape. 3. The QRS complex is less than 0.12 seconds wide. 4. The polarity of the T wave is opposite that of the QRS complex. 5. Most ventricular dysrhythmias have no apparent P wave.

2, 4, 5 Ventricular dysrhythmias arise from ectopic foci in the ventricles. Because the stimulus depolarizes the ventricles in a slower, abnormal way, the QRS complex appears widened and has a bizarre shape. The QRS complex is wider than 0.12 seconds and often wider than 0.16 seconds. The polarity of the T wave is opposite that of the QRS complex. Depolarization from abnormal ventricular beats rarely activates the atria in a retrograde fashion. Therefore most ventricular dysrhythmias have no apparent P waves. However, if a P wave is present, it is usually seen in the T wave of the following beat, or it has no relationship to the QRS complex and is dissociated from the ventricular rhythm. Ventricular dysrhythmias can be life-threatening; thus, fast recognition and intervention is imperative.

While receiving a blood transfusion, a client develops flank pain, chills, and fever. What type of transfusion reaction does the nurse conclude that the client probably is experiencing? 1. Allergic 2. Pyrogenic 3. Hemolytic 4. Anaphylactic

3 A hemolytic transfusion reaction results from a recipient's antibodies that are incompatible with transfused red blood cells; it is called a type II hypersensitivity. The clinical findings are a result of red blood cell hemolysis, agglutination, and capillary plugging. An allergic transfusion reaction is the result of an immune sensitivity to foreign serum protein; it is called a type I hypersensitivity and associated clinical findings include urticaria, wheezing, dyspnea, and shock. Bacterial pyrogens are present in contaminated blood and can cause a febrile transfusion reaction; associated clinical findings include fever and chills, but not flank pain. There is no transfusion reaction called an anaphylactic transfusion reaction; anaphylaxis may occur with an allergic transfusion reaction.

When caring for a client who has hyponatremia, the nurse would monitor for which of the following? 1. Increased urine output 2. Deep rapid respirations 3. Decreased specific gravity 4. Distended neck veins

3 A normal sodium level is between 135 and 145 mEq/L of sodium. Hyponatremia occurs when the sodium level falls below 135 mEq/L. Decreased specific gravity may be caused from aldosteronism, excessive fluid intake, diabetes insipidus central, diabetes insipidus nephrogenic, kidney failure, renal tubular necrosis, or severe kidney infection (pyelonephritis). Symptoms are nonspecific and can include mental changes, headache, nausea and vomiting, tiredness, muscle spasms, and seizures. Increased urine output, deep rapid respirations, and distended neck veins are not associated with hyponatremia.

A client with asthma is being taught how to use a peak flow meter to monitor how well the asthma is being controlled. What should the nurse instruct the client to do? 1. Perform the procedure once in the morning and once at night 2. Move the trunk to an upright position and then exhale while bending over 3. Inhale completely and then blow out as hard and as fast as possible through the mouthpiece 4. Place the mouthpiece between the lips and in front of the teeth before starting the procedure

3 A peak flow meter measures the peak expiratory flow rate, the maximum flow of air that can be forcefully exhaled in one second; this monitors the pulmonary status of a client with asthma. The peak flow measurement should be done daily in the morning before the administration of medication or when experiencing dyspnea. The client should be standing or sitting upright. Placing the mouthpiece between the lips and in front of the teeth before starting the procedure will interfere with an accurate test; the mouthpiece should be in the mouth between the teeth with the lips creating a seal around the mouthpiece.

The nurse recognizes that an early finding that indicates that a client is hypertensive is: 1. An extended Korotkoff sound 2. An irregular pulse of 92 beats per minute 3. A diastolic blood pressure that remains greater than 90 mm Hg 4. A throbbing headache over the left eye when arising in the morning

3 A sustained diastolic pressure exceeding 90 mm Hg reflects pathology and indicates hypertension. An extended Korotkoff sound is unrelated to hypertension. An irregular pulse of 92 beats per minute reflects the heart rate and rhythm, not the pressure within the arteries. Initially hypertension usually is asymptomatic; although headaches can be associated with hypertension, there are other causes of headaches.

The plan of care for a postoperative client who has developed a pulmonary embolus includes monitoring and bed rest. The client asks why all activity is restricted. The nurse's response is based on the principle that bed rest: 1. Prevents the further aggregation of platelets 2 Enhances the peripheral circulation in the deep vessels 3 Decreases the potential for further dislodgment of emboli 4 Maximizes the amount of blood available to damaged tissues

3 Activity may encourage the dislodgment of more microemboli. Bed rest may enhance platelet aggregation and the formation of thrombi because of venous stasis. Bed rest supports venous stasis, rather than enhanced circulation. Bed rest supports venous stasis rather than the circulation of blood to damaged tissues.

What is the nurse primarily attempting to prevent when caring for a client in the initial stages of chronic lymphocytic leukemia (CLL)? 1. Injury 2 Fatigue 3 Infection 4 Cachexia

3 Although lymphocytosis is always present, defects in humoral and cellular immunity increase the risk for infection. Injury becomes an issue later in the disease when thrombocytopenia may develop. Fatigue becomes an issue later in the disease when anemia may develop. Although excessive weight loss is a concern, it does not pose the same threat as infection for clients with CLL.

The nurse is assessing a client for signs of right ventricular failure. What should the nurse expect if this occurs? 1. Slowed pulse rate 2. Pleural friction rub 3. Neck vein distention 4. Increasing hypotension

3 Neck vein distention is caused by hypervolemia and pulmonary hypertension. The pulse is likely to be rapid and bounding. Pleural friction rub is present in pleurisy, not heart failure. Hypertension, not hypotension, will occur because of hypervolemia.

When performing an assessment on a client with probable acute lymphoblastic leukemia (ALL), the clinical manifestation the nurse expects to be present is: 1. Alopecia 2. Insomnia 3. Ecchymosis 4. Splenomegaly

3 Bleeding tendencies occur because of bone marrow suppression and rapidly proliferating leukocytes. There is no change in hair growth in the absence of chemotherapy. The client more likely will be sleep

A client with a cardiac dysrhythmia is receiving digoxin (Lanoxin) and verapamil (Calan). Because of the combined effect of these two medications, the nurse assesses the client for: 1. Physical agitation 2. Reflex stimulation 3. Myocardial depression 4. Respiratory stimulation

3 Both digoxin and verapamil decrease cardiac impulse conduction, with resultant depression of the myocardium; verapamil decreases conduction at the sinoatrial (SA) and atrioventricular (AV) nodes, which may cause bradycardia, AV block, and cardiac arrest. Digoxin and verapamil together do not cause agitation. Side effects of verapamil include fatigue and depression, not agitation. Digoxin and verapamil do not influence the reflexes of the body. Digoxin and verapamil do not influence respirations.

In addition to atrial fibrillation, what ventricular rhythm exhibited by a client does the nurse determine may be converted to a sinus rhythm by cardioversion? 1. Standstill 2. Fibrillation 3. Super ventricular tachycardia 4. Frequent premature complexes

3 Cardioversion involves administration of precordial shock, which is synchronized with the R wave to interrupt the heart rate. It is used for atrial fibrillation, supraventricular tachycardia, and ventricular tachycardia with a pulse when pharmaceutical preparations fail. The heart is stopped by the electric stimulation, and it is hoped that the sinoatrial (SA) node will take over as pacemaker. Because there are no R waves, cardioversion should not be done. Premature ventricular complexes suggest an irritable myocardium and generally respond well to antidysrhythmic agents.

A nurse is caring for a client eight hours after surgery. The client's portable wound drainage device is half full of drainage. After emptying the drainage collection chamber, the nurse creates negative pressure in the system by: 1. Attaching the device to a wall suction unit 2. Milking the tubing toward the suction device 3. Compressing the device while closing the air plug 4. Keeping the device in a position lower than the site of insertion

3 Compressing the device expels air in the unit, and closing the plug while it is compressed reestablishes the closed system and creates negative pressure. A portable suction device usually is not attached to a mechanical suction machine. Milking the tubing promotes patency but will not create negative pressure. Although a portable wound drainage container is kept below the level of the insertion site, which facilitates drainage by gravity, this will not create negative pressure in the system.

A health care provider prescribes daily sputum specimens to be collected from a client. When is the most appropriate time for the nurse to collect these specimens? 1. After activity 2. Before meals 3. On awakening 4. Before a respiratory treatment

3 During sleep, mucous secretions in the respiratory tract move slowly toward the throat. On awakening, increased ciliary motion raises these secretions more vigorously, thus facilitating expectoration and the collection of sputum specimens. Although activity mobilizes secretions, no secretions may be present at the time of activity; sputum is most plentiful upon arising. The sputum may leave an unpleasant taste in the mouth, which may interfere with appetite. Sputum more likely would be collected after, not before, a respiratory treatment, because this mobilizes secretions.

A client has a persistent productive cough that becomes blood tinged. A needle biopsy is scheduled. The client tells the nurse, "During the procedure, a needle will be inserted into my back to collapse my lung." Which is the most appropriate response by the nurse? 1. "Your perception of the diagnostic test is incorrect." 2. "I will ask the primary health care provider to clarify the diagnostic procedure." 3. "Tell me more about the conversation you had with your health care provider." 4. "The procedure will be fast so that you will experience minimal discomfort."

3 Exploration and collection of data are important parts of the therapeutic process; anxiety, fear, and depression can influence understanding. The response "Your perception of the diagnostic test is incorrect" will put the client on the defensive. Instructing the client to ask the health care provider to clarify the procedure is not the priority; at this point, the nurse should collect more data. The response "The procedure will be fast so that you will experience minimal discomfort" is false reassurance.

What response provides evidence that a client with chronic obstructive pulmonary disease (COPD) understands the nurse's instructions about an appropriate breathing technique? 1. Inhales through the mouth. 2. Increases the respiratory rate. 3. Holds each breath for a second at the end of inspiration. 4. Progressively increases the length of the inspiratory phase.

3 Holding each breath for a second at the end of inspiration allows added time for gaseous exchange at alveolar capillary beds. Inhalation should occur through the nose to moisten, filter, and warm the air. Increasing the respiratory rate does not allow for prolonged expirations, thus decreasing the effectiveness of respirations. The expiratory phase should be lengthened, and exhalation should occur through pursed lips to prevent alveolar collapse.

A client's respiratory status deteriorates, and endotracheal intubation and positive pressure ventilation are instituted. What is the nurse's most immediate intervention at this time? 1. Prepare the client for emergency surgery. 2. Facilitate the client's verbal communication. 3. Assess the client's response to the interventions. 4. Maintain sterility of the ventilation system that is being used.

3 If a client is not responding to interventions, the plan must be changed to support respiration. Preparing the client for emergency surgery is presumptive; there are insufficient data to conclude that surgery is necessary. Endotracheal intubation does not permit verbal communication. Maintaining sterility of the ventilation system that is being used is important, but it is not the priority.

Metoprolol (Lopressor) is prescribed for a client. The nurse should question the prescription if the client has which diagnosis? 1. Hypertension 2. Angina pectoris 3. Sinus bradycardia 4. Myocardial infarction

3 Metoprolol is a beta blocker; it decreases the heart rate and thus is contraindicated with bradycardia. Metoprolol is an antihypertensive agent. By reducing cardiac output, metoprolol reduces myocardial oxygen consumption, which helps prevent ischemia and anginal pain.

WBC 10,000 mm3 Hbg 11 gm/dl Hct 34% INR 2.5 T 100 F P 100, regular rhythm R 24 B/P 176/96 Respirations 24, using pursed lip breathing, pulse bounding, face appeared flush; reports a headache and feeling dizzy. A nurse reviews the laboratory test results of a client with emphysema who is recovering from a myocardial infarction. The nurse obtains the client's vital signs and performs a physical assessment. Which prescribed medication should the nurse consider the priority at this time? 1. Albuterol (Proventil) 2. Warfarin (Coumadin) 3. Metoprolol (Lopresor) 4. Acetaminophen (Tylenol)

3 Metoprolol is indicated for the control of a blood pressure of 176/96, which increases the cardiac workload and may lead to myocardial ischemia. Albuterol, a bronchodilator, is not the priority at this time; its administration may be delayed until the blood pressure is controlled because a side effect of this drug is hypertension. Warfarin, an anticoagulant, is not the priority at this time; the international normalized ratio (INR) is within the therapeutic range of 2 to 3. A slight temperature increase is an expected response to a myocardial infarction; however, the administration of the antipyretic acetaminophen is not the priority.

A client with a coronary occlusion is experiencing chest pain and distress. The nurse administers oxygen to: 1. Prevent dyspnea. 2. Prevent cyanosis. 3. Increase oxygen concentration to heart cells. 4. Increase oxygen tension in the circulating blood.

3 Oxygen increases the transalveolar oxygen gradient, which improves the efficiency of the cardiopulmonary system. This enhances the oxygen supply to the heart. Increased oxygen to the heart cells will improve cardiac output, which may or may not prevent dyspnea. Pallor, not cyanosis, usually is associated with myocardial infarction. Although increasing oxygen tension in the circulating blood may be true, it is not specific to heart cells.

The nurse's physical assessment of a client with heart failure reveals tachypnea and bilateral crackles. What is the priority nursing intervention? 1. Initiate oxygen therapy 2. Obtain a chest x-ray film immediately 3. Place client in a high-Fowler position 4. Assess the client for a pleural friction rub

3 Placing the client in a high-Fowler positionpromotes lung expansion and gas exchange; it also decreases venous return and cardiac workload. Initiating oxygen therapy may be done, but positioning should be done first because it will have an immediate effect. Time is needed to set up the system for the delivery of oxygen. Maintaining adequate oxygen exchange is the priority; an x-ray film can be obtained, but after breathing is supported. A friction rub is related to inflammation of the pleura, not to heart failure.

After abdominal surgery a client should be encouraged to turn from side to side and to engage in deep-breathing exercises. The nurse explains that these activities are essential to prevent: 1. Metabolic acidosis 2 Metabolic alkalosis 3 Respiratory acidosis 4 Respiratory alkalosis

3 Shallow respirations, bronchial tree obstruction, and atelectasis compromise gas exchange in the lungs; an elevated carbon dioxide level leads to acidosis. Metabolic acidosis is caused by a loss of bicarbonate from the lower gastrointestinal tract, which is associated with diarrhea. Metabolic alkalosis is caused by excessive loss of hydrogen ions from gastric decompression or excessive vomiting. Respiratory alkalosis is caused by increased expiration of carbon dioxide, a component of carbonic acid.

What must the nurse determine before discontinuing airborne precautions for a client with pulmonary tuberculosis? 1. Client no longer is infected. 2. Tuberculin skin test is negative. 3. Sputum is free of acid-fast bacteria. 4. Client's temperature has returned to normal.

3 The absence of bacteria in the sputum indicates that the disease can no longer be spread by the airborne route. Treatment is over an extended period; eventually the client may not have an active disease, but still remains infected. Once an individual has been infected, the test will always be positive. The client's temperature returning to normal is not evidence that the disease cannot be transmitted.

A client who is receiving atenolol (Tenormin) for hypertension frequently reports feeling dizzy. What effect of atenolol should the nurse consider may be responsible this response? 1. Depleting acetylcholine 2. Stimulating histamine release 3. Blocking the adrenergic response 4. Decreasing adrenal release of epinephrine

3 The beta adrenergic blocking effect of atenolol (Tenormin) decreases the heart's rate and contractility; it may result in orthostatic hypotension and decreased cerebral perfusion, causing dizziness. Depleting acetylcholine is not an action of atenolol. Stimulating histamine release is not an action of atenolol. Decreasing adrenal release of epinephrine is not an action of atenolol.

A client diagnosed with multiple myeloma has been given a poor prognosis. After discharge, the client plans to travel on an airplane and attend sporting events with friends and family. The nurse prepares a discharge teaching plan for this client and includes: 1. Eliminating travel plans to combat anemia-related fatigue 2. Reinforcing a positive mental attitude to improve prognosis 3. Preventing infection; the client is at risk for leukopenia 4. Restricting fluid intake; the client is at risk for congestive heart failure

3 The bone marrow is impaired with multiple myeloma; the effectiveness of white blood cells and immunoglobulins is reduced, which increases susceptibility to bacterial infections. Travel can be accomplished with careful planning and adequate rest periods. Although a positive mental attitude can contribute to quality of life and may even extend life, generally it does not change the prognosis. The client is encouraged to drink plenty of fluids to help dilute the Bence Jones protein fragments in the urine, which may help prevent kidney damage.

The nurse is caring for a client who had a wedge resection of a lobe of the lung and now has a chest tube with a three-chamber underwater drainage system in place. The nurse considers that the main purpose of the third chamber of the underwater drainage system is to: 1. Act as a drainage container 2 Provide an airtight water seal 3 Control the amount of suction 4 Allow for escape of air bubbles

3 The first chamber collects drainage; the second chamber provides for the underwater seal; the third chamber controls the amount of suction. The first chamber, not the third chamber, collects drainage. The second chamber, not the third chamber, provides an underwater seal. Although allowing for escape of air bubbles occurs in a three-chamber system, the purpose of the third chamber is to control suction.

A nurse is teaching a client with hypertension about a sodium-restricted diet. What information should the nurse emphasize? 1. Using salt-free natural seasonings can taste the same as salt. 2. Desiring the taste for salt is inherent but can be overcome with practice. 3. Liking the taste of table salt is learned but it is not a biological necessity. 4. Substituting table salt with potassium chloride can be done freely.

3 The taste for salt is learned from habitual use and can be unlearned or reduced with health improvement motivation and creative salt-free food preparation. Substitutes do not taste the same as salt. The taste for salt is learned. Using salt substitutes containing potassium chloride may be unsafe; excessive use can produce abnormally high serum potassium levels.

A nurse concludes that the simvastatin (Zocor) being administered to a client is effective. A decrease in what clinical finding supports this conclusion? 1. International normalized ratio (INR) 2. Heart rate 3. Triglycerides 4. Blood pressure

3 Therapeutic effects of simvastatin include decreased levels of serum triglycerides, low-density lipoprotein (LDL), and cholesterol. INR is not related to simvastatin; it is a measure used to evaluate blood coagulation. Heart rate and blood pressure are not related to simvastatin.

When auscultating a client's chest, the nurse hears swishing sounds of normal breathing. The nurse should document: 1. Adventitious sounds 2 Fine crackling sounds 3 Vesicular breath sounds 4 Diminished breath sounds

3 These are normal respiratory sounds heard on auscultation as inspired air enters and leaves the alveoli. Adventitious is the general term for all abnormal breath sounds. Crackles heard at the end of an inspiration are associated with pulmonary edema. Diminished breath sounds are evidence of a reduction in the amount of air entering the alveoli; this usually is caused by obstruction or consolidation.

A client with a history of heart failure and hypertension is admitted with reports of syncope. Which prescribed medication should the nurse prepare to administer based on the ECG rhythm strip image? (sinus bradycardia) 1. Digoxin (Lanoxin) 2. Enalapril (Vasotec) 3. Atropine 4. Metoprolol (Lopressor)

3 This rhythm strip reflects sinus bradycardia. Sinus bradycardia has PQRST complexes within acceptable limits, but the rate is less than 60 beats per minute. In this strip the PR interval is 0.16, the rhythm is regular, and the rate is 40 beats per minute. Atropine, an anticholinergic that increases the heart rate, is administered when the heart rate is so slow that it causes symptoms. Digoxin is a cardiac glycoside that slows the heart rate. Enalapril is an angiotensin-converting enzyme (ACE) inhibitor that slows the heart rate. Metoprolol is a beta blocker that slows the heart rate.

When a client is experiencing hypovolemic shock with decreased tissue perfusion, the nurse expects that the body initially attempts to compensate by: 1. Producing less antidiuretic hormone (ADH) 2 Producing more red blood cells 3 Maintaining peripheral vasoconstriction 4 Decreasing mineralocorticoid production

3 With shock, arteriolar vasoconstriction occurs, raising the total peripheral vascular resistance and shifting blood to the major organs. With shock, more ADH is produced to promote fluid retention, which will elevate the blood pressure. Although producing more red blood cells is a response to hypoxia, peripheral vasoconstriction is a more effective compensatory mechanism. With shock the mineralocorticoids increase to promote fluid retention, which elevates the blood pressure.

The nurse is making rounds on a client who has developed severe bone marrow depression after receiving chemotherapy for cancer. Which of these actions by the nurse is appropriate? Select all that apply. 1. Monitor for signs of alopecia. 2. Encourage an increase in fluids. 3. Wash hands before entering the client's room. 4. Advise use of a soft toothbrush for oral hygiene. 5. Report an elevation in temperature immediately. 6. Encourage the client to eat raw, fresh fruits and vegetables.

3, 4, 5 It is essential to prevent infection in a client with severe bone marrow depression; thorough hand-washing before touching the client or client's belongings is important. Thrombocytopenia occurs with most chemotherapy treatment programs; using a soft toothbrush helps prevent bleeding gums. Any temperature elevation in a client with neutropenia must be reported to the health care provider immediately as it may be a sign of infection. Although alopecia does occur with chemotherapy, it is not related to bone marrow suppression. Increasing fluids will neither reverse bone marrow suppression nor stimulate hematopoiesis. This is not related to bone marrow suppression. Clients who have severe bone marrow depression must avoid eating raw fruits and vegetables, and undercooked meat, eggs, and fish to avoid possible exposure to microbes.

A nurse is teaching a client who had a myocardial infarction about the prescribed 1500-calorie, 2-gram sodium, weight-reducing diet. Which low-sodium, low-calorie nutrients should the nurse recommend that the client include in the diet? Select all that apply. 1. Lean steak 2. Celery sticks 3. Baked chicken 4. Tuna fish salad 5. Mashed potatoes

3, 5 Baked chicken is low in calories and sodium. Mashed potatoes are low in sodium and calories. Beef is high in calories. Celery sticks are high in sodium. Canned tuna fish is high in sodium.

When planning a citywide national homeland security disaster preparedness drill, a nurse is assigned to the triage committee. Place the following criteria in an order that reflects the most efficient triage plan for an actual mass casualty incident (MCI). 1. Severe wrist sprain 2. Crushing head injury 3. Tension pneumothorax 4. Laceration of thigh muscle 5. Compound fracture of femur

3, 5, 4, 1, 2 Critically ill clients who can survive with care are categorized as "immediate" and are treated first; they receive a code of "Red." In a major disaster, the exact opposite of what is done in a nondisaster situation occurs. The object is to get the greatest number of survivors treated fast. A tension pneumothorax involves increasing positive pressure in the pleural space which leads to lung collapse. An inability to breathe can quickly lead to death if not treated immediately. Open fractures of the long bones are also categorized as "Red" because if untreated, infection and bleeding are life threatening complications. This, however, does not pose the immediate threat that a tension pneumothorax does. Those who are injured but for whom care can be postponed for a few hours are categorized as "delayed" and treated second; they receive a code of "Yellow." Treatment of vascular injuries without evidence of shock can be delayed. Those who have minor injuries are categorized as "minimal" and are treated third; they receive a code of "Green." A sprain is not life-threatening; care can be delayed for an extended period. Those who are critically ill and have little or no chance of survival are categorized as "expectant" and are treated last; they receive a code of "Black." A crushing head injury causes severe brain injury; chances of survival are unlikely.

The health care provider prescribes aminophylline 500 mg in 500 mL normal saline (NS) to infuse over 4 hours for a client who has been admitted for an acute exacerbation of chronic obstructive pulmonary disease (COPD). The tubing delivers 60gtt/min. The nurse should set the intravenous (IV) pump to administer how many milliliters per hour? 1. 60 mL/hr 2 65 mL/hr 3 120 mL/hr 4 125 mL/hr

4 500 ml/ 4 hr = 125 ml/hr

A health care provider prescribes one unit of whole blood for a client after gastrointestinal surgery. What is an important nursing action when administering blood? 1. Maintain patency of the intravenous catheter with dextrose solution. 2. Warm the blood to body temperature to prevent chilling the client. 3. Draw a blood sample from the client before each unit is transfused. 4. Run the blood at a slower rate during the first few minutes of the transfusion.

4 A slow rate provides time to recognize a transfusion reaction that is developing before too much blood has been administered. Dextrose solution will cause lysis of the red blood cells (RBCs); saline must be used. Warming the blood to body temperature may cause clotting and hemolysis. Blood samples may be drawn after, not before, a transfusion, but this is not routinely done.

A client is admitted to the hospital for replacement of the mitral valve. The primary purpose of the nurse checking the pulses in the client's legs frequently after surgery is to detect: 1. Atrial fibrillation 2. Postsurgical bleeding 3. Arteriovenous shunting 4. Peripheral thrombophlebitis

4 Because blood pools in the extremities, there is an increased hazard of peripheral emboli in clients who have received a mitral valve replacement. Atrial fibrillation is assessed by cardiac monitor interpretation and by the presence of a pulse deficit. The nurse would need to compare a peripheral pulse with the apical pulse. Bleeding is detected by checking the wound dressing and observing for signs of shock (e.g., lowered blood pressure, tachycardia, restlessness). Arteriovenous shunting is not a danger after mitral valve replacement.

A client with a bundle branch block is on a cardiac monitor. What ECG change should the nurse identify on the client's cardiac monitor? 1. Sagging ST segments 2 Absence of P wave configurations 3 Inverted T waves following each QRS complex 4 Widening of QRS complexes to a minimum of 0.12 second

4 Bundle branch block interferes with the conduction of impulses from the atrioventricular node to the ventricle supplied by the affected bundle. Conduction through the ventricles is delayed, as evidenced by a widened QRS complex. Changes in the T waves or ST segments usually occur as a result of cardiac damage. P waves, produced when the sinoatrial (SA) node fires to begin a cycle, are present in bundle branch block.

A client had a pneumonectomy. For which postoperative complication that is specific to this type of surgery should the nurse assess this client? 1. Brain attack 2. Renal failure 3. Internal bleeding 4. Cardiac overload

4 Cardiac overload can be caused by the loss of the large vascular lung or a mediastinal shift. A brain attack is not unique to a pneumonectomy. Renal failure is not unique to a pneumonectomy. Internal bleeding is not unique to a pneumonectomy.

Enoxaparin (Lovenox) 40 mg subcutaneously daily is prescribed for a client who had abdominal surgery. The nurse explains that the medication is given to: 1. Control expected postoperative fever 2. Provide a constant source of mild analgesia 3. Limit the inflammatory response associated with surgery 4. Provide prophylaxis against postoperative thrombus formation

4 Enoxaparin (Lovenox), a low-molecular-weight heparin, prevents the conversion of fibrinogen to fibrin and prothrombin to thrombin by enhancing the inhibitory effects of antithrombin III. Enoxaparin is not an antipyretic. Enoxaparin is not an analgesic. Enoxaparin is not an antiinflammatory drug.

A nurse works with a large population of immigrant clients and is concerned about the debilitating effects of influenza. Which action is the first line of defense against an emerging influenza pandemic? 1. Complying with quarantine measures 2. Instituting strict international travel restrictions 3. Seeking aid from the international public health community 4. Reporting surveillance findings to appropriate public health officials

4 Honesty and openness are essential to understanding the extent of the problem so that an appropriate local and global response can be mobilized to limit emerging pandemics. While complying with quarantine measures helps, it can only be done in response to detection and reporting of the presence of an emerging health problem. In response to the severe acute respiratory syndrome (SARS) epidemic of 2002, the International Air Transport Association began work to standardize procedures that address passenger screening and the accurate and quick tracking of passenger travel.

A client with chronic obstructive pulmonary disease has increased hemoglobin and hematocrit levels. The nurse concludes that the altered blood levels are caused by: 1. Increased leukocyte development in response to infection 2. Decreased extracellular fluid volume secondary to infection 3. Decreased red blood cell proliferation because of hypercapnia 4. Increased erythrocyte production as a result of chronic hypoxia

4 Hypoxia stimulates production of large quantities of erythrocytes in an attempt to compensate for the lack of oxygen. White blood cell production increases with infection; infection is not the cause of the increase in the hemoglobin and hematocrit. There is a loss of extracellular fluid in an acute infection with a fever; however, in a chronic condition, this fluid is replenished and the hematocrit usually is unaffected. Hypercapnia is an increase in PCO2 in extracellular fluid; this does not have a direct effect on the hemoglobin and hematocrit levels.

The nurse is interpreting the client's rhythm strip and finds that the P and QRS waves are consistent, with a P wave preceding every QRS complex. The PR interval is 0.26 seconds long. The rate is 64 beats per minute. The nurse interprets this rhythm as: 1. Normal sinus rhythm (NSR). 2. Sinus rhythm with second degree atrioventricular (AV) block. 3. Complete heart block. 4. Sinus rhythm with first degree AV block.

4 In first degree block, P and QRS waves are consistent in shape. P waves are small and rounded. A P wave precedes every QRS complex, which is followed by a T wave. PR interval is prolonged and is greater than 0.20 seconds. QRS complex and QT/QTc measurements are normal. NSR reflects normal conduction of the sinus impulse through the atria and ventricles. Atrial and ventricular rates are the same and range from 60 to 100 beats per minute. Rhythm is regular or essentially regular. PR interval is 0.12 to 0.20 seconds.

A client's cells are deprived of oxygen during a cardiac arrest. What medication should the nurse be prepared to administer? 1. Regular insulin 2. Calcium gluconate 3. Potassium chloride 4. Sodium bicarbonate

4 In the absence of oxygen, the body derives its energy anaerobically; this results in buildup of lactic acid. Sodium bicarbonate, an alkaline drug, will help neutralize the acid, raising the pH. Insulin is used to treat diabetes; it lowers blood sugar by facilitating transport of glucose across cell membranes. Calcium gluconate is used to treat hypocalcemia. Although potassium is essential for cardiac function, it will not correct acidosis. With acidosis, serum hydrogen ions will exchange with intracellular potassium, leading to a temporary hyperkalemic state; therefore, potassium chloride is contraindicated until acidosis is corrected.

A nurse is preparing to insert a nasogastric tube. During insertion, which response indicates that the client is experiencing difficulty? 1. Gagging 2 Discomfort 3 Flushed face 4 Inability to speak

4 Inability to speak indicates that the tube may be in the airway. Discomfort is a normal response. Gagging may occur as the tube passes from the nasal passage through the pharynx. Flushing may result if the client attempts to fight passage of the tube.

On a client's admission to a rehabilitation unit, the nurse gives the client, who is not immunocompromised, a purified protein derivative (PPD) of tuberculin to test for tuberculosis. Which client reaction indicates a positive response? 1. 5 mm erythema with no induration 2. No erythema with 3 mm induration 3. 7 mm erythema with 5 mm induration 4. 5 mm erythema with 10 mm induration

4 Induration of 10 mm or greater is a positive test result in clients with effective immune systems. Erythema without induration is not considered a positive test result. Induration of 0 to 4 mm is not considered a positive test result. Induration of 7 mm erythema with 5 mm is considered a positive reaction in individuals who are immunocompromised or at high risk for tuberculosis.

The nurse is planning nutritional education for a client with lower extremity arterial disease (LEAD). What diet modifications should the nurse include? 1. Decreasing both fluid and sodium intake 2. Increasing both calcium and potassium intake 3. Increasing both vitamin E and refined grain intake 4. Decreasing both cholesterol and saturated fat intake

4 Lower extremity arterial disease frequently is accompanied by generalized atherosclerosis; decreasing both cholesterol and saturated fat intake will help decrease lipid buildup on artery walls. Decreasing both fluid and sodium intake are inappropriate dietary modifications; this client does not have edema. Increasing both calcium and potassium is not appropriate for the client's condition because it may alter the client's electrolyte balance. Recent research indicates that supplemental vitamin E can precipitate cardiac problems and only should be taken when prescribed by a health care provider who can monitor the client's ongoing status. Increasing grain intake will add calories and may contribute to unnecessary weight gain.

The nurse helps a client create a list of appropriate food choices to maintain a 2-gram sodium diet that recently has been prescribed for the client. The nurse also assesses the client's cooking habits. The client tells the nurse that, at home, all food is cooked without salt. The nurse concludes that further teaching is needed when the client places what food items on the list of appropriate food choices? 1. Soft-cooked egg, salt-free toast, jelly, and skim milk 2 Baked chicken, boiled potatoes, broccoli, and coffee 3 Fillet of sole, baked potato, lettuce and tomato salad, fresh fruit cup, and milk Correct4 Mixed fruit salad bowl with cottage cheese, crackers, celery, sweet pickles, and tea

4 Mixed fruit salad bowl with cottage cheese, crackers, celery, sweet pickles, and teahas the highest sodium content. Soft-cooked egg, salt-free toast, jelly, skim milk, baked chicken, boiled potatoes, broccoli, coffee, fillet of sole, baked potato, lettuce and tomato salad, and fresh fruit cup are low in sodium. Test-Taking Tip: Do not read information into questions, and avoid speculating. Reading into questions creates errors in judgment.

A client with a history of rheumatic fever and a heart murmur reports gaining weight in spite of nausea and anorexia. The client also reports shortness of breath several times each day and when performing minor tasks. Which additional information should the nurse obtain? 1. Retrospective 24-hour calorie count 2 Elimination pattern during the last 30 days 3 Complete gynecological and sexual history 4 Presence of a cough and pulmonary secretions

4 Presence of a cough and pulmonary secretions, in addition to a history of rheumatic fever, require an assessment for other cardiopulmonary problems. Anorexia and weight gain do not indicate a nutritional problem but a fluid balance problem. Loss of appetite in conjunction with shortness of breath and the history of rheumatic fever make gastrointestinal symptoms secondary in importance. There is no reason to investigate the gynecological and sexual history in relation to the current problem.

A health care provider prescribes an antihypertensive medication. Which over-the-counter medication should the nurse teach the client to avoid because it has the potential to counteract the effect of the antihypertensive? 1. Omeprazole (Prilosec) 2 Acetaminophen (Tylenol) 3 Docusate sodium (Colace) 4 Pseudoephedrine (Sudafed)

4 Pseudoephedrine has a pressor effect that may counteract antihypertensive medications, causing an increase in blood pressure. Omeprazole does not interact with antihypertensives. However, it can increase the action of phenytoin, digoxin, clopidrogrel (Plavix), and cyclosporine. Acetaminophen does not have to be avoided when receiving an antihypertensive. Docusate sodium does not have to be avoided when receiving an antihypertensive.

A client is admitted with stage 2 hypertension. What diastolic pressure does the nurse consider to be consistent with this diagnosis? 1. 80 to 89 mm Hg 2. 90 to 99 mm Hg 3. Less than 79 mm Hg 4. More than 100 mm Hg

4 Stage 2 hypertension is identified by a systolic pressure of 160 to 179 mm Hg or a diastolic pressure more than 100 mm Hg. Diastolic pressure 80 to 89 mm Hg reflects prehypertension. Diastolic pressure 90 to 99 mm Hg reflects stage 1 hypertension. Less than 79 mm Hg reflects a diastolic blood pressure that is considered within the desirable range.

A client comes to the clinic for a physical and asks to be tested for acquired immunodeficiency syndrome (AIDS). Which test should the nurse explain will be used for the initial screening for HIV? 1. CD4 T cell count 2. Western blot test 3. Polymerase chain reaction test 4. Enzyme-linked immunosorbent assay (ELISA)

4 The ELISA is the first screening test done to detect serum antibodies that bind to human immunodeficiency virus (HIV) antigens on test plates. The CD4 T cell count is not a screening test; it is done to monitor the progression of HIV infection and response to treatment. The Western blot test is not done first; the Western blot is done to validate repeatedly reactive ELISA results. The polymerase chain reaction test is not an initial screening test; it is done when there are consistently inconclusive test results with previous screening tests.

A nurse prepares a client for insertion of a pulmonary artery catheter. What information can be obtained from monitoring the pulmonary artery pressure? 1. Stroke volume 2. Venous pressure 3. Coronary artery patency 4. Left ventricular functioning

4 The catheter is placed in the pulmonary artery. Information regarding left ventricular function is obtained when the catheter balloon is inflated. Information on stroke volume, the amount of blood ejected by the left ventricle with each contraction, is not provided by a pulmonary catheter. Although a central venous pressure reading can be obtained with the pulmonary catheter, it is not as specific as a pulmonary wedge pressure, which reflects pressure in the left side of the heart. The patency of the coronary arteries usually is evaluated by cardiac catheterization.

To help reduce a client's risk factors for heart disease, what should the nurse include in the client's teaching plan? 1. Avoid eating between meals 2. Limit unsaturated fats in the diet 3. Decrease the amount of fat-binding fiber 4. Increase the quantity of complex carbohydrates

4 The fiber component of complex carbohydrates helps bind and eliminate dietary cholesterol and fosters growth of intestinal microorganisms to break down bile salts and release the cholesterol component for excretion. It is what, not when, the client eats that is important. Saturated fats should be decreased. Fat-binding fiber should be increased.

A client weighing 125 kilograms (275 pounds), is considered to be in septic shock when the mean arterial pressure is less than 65 mm Hg or the systolic blood pressure is less than 90 mm Hg after receiving how many liters of intravenous crystalloids? 1. 0.5 L 2. 1 L 3. 2 L 4. 5 L

4 This question requires the learner to apply the formula for defining septic shock to the scenario described. A septic client is considered to be in septic shock if the client remains hypotensive, as defined in the stem of the question, in spite of receiving 20-40 mL/kg of crystalloids, making 5 L the correct option.

A nurse is caring for a client who just had major abdominal surgery. What client responses indicate the possibility of developing a superficial venous thrombosis? Select all that apply. 1. Pitting edema of the ankle 2. Reddened area at the ankle 3. Pruritus on the side of the calf 4. Tender area in the posterior lower leg 5. Warmth along the course of the involved vessel

4, 5 Thrombophlebitis, not uncommon after abdominal surgery, is inflammation of a vein; it is associated with the formation of a clot (thrombus) in a vein in the leg. Findings associated with thrombophlebitis include pain, redness, swelling, and heat. Warmth along the course of the involved vessel is related to the inflammatory process accompanying the thrombus. Although swelling accompanies thrombophlebitis, it is not a pitting edema. Thrombophlebitis usually is located in the area of the calf, not over a bony prominence. Itching is not a symptom of phlebitis.

A client is experiencing dyspnea. In which position should the nurse place the client? 1. Sims 2. Supine 3. Orthopneic 4. Trendelenburg

The orthopneic position refers to sitting up and leaning slightly forward, which lowers the diaphragm, allowing the lungs more room for expansion. Horizontal positions, such as Sims and supine, do not allow the gravitational effect on the diaphragm and thus do not maximize air exchange. The Trendelenburg position forces the diaphragm up toward the lungs, thereby interfering with lung expansion.

A client is admitted to the coronary intensive care unit. What is the first step when the nurse is developing a discharge teaching plan for this client? 1. Identifying the client's needs 2. Formulating the client's desired outcomes 3. Exploring the client's community resources 4. Assessing the client's personal support system

1 For teaching to be meaningful, the client must have a need to learn; also, readiness to learn is part of this assessment. The nurse determines expected outcomes depending on mutually desired goals; also, this is not the first step when developing a discharge teaching plan. Exploring the client's community resources is not the initial step; assessment of learning needs comes first. Assessing the client's personal support system is not the initial intervention; the client's needs must be assessed first.

A client sustains a crushing injury to the lower left leg, and a below-the-knee amputation is performed. For which common clinical manifestations of a pulmonary embolus should the nurse assess this client? Select all that apply. 1. Unilateral chest pain 2. Acute onset of dyspnea 3. Pain in the residual limb 4. Absence of the popliteal pulse 5. Blanching of the affected extremity

1, 2 Emboli can occur with crushing injuries of the extremities. Lodging of a thrombus in the pulmonary system results in a lack of oxygen to pulmonary tissues, causing localized pain. Lodging of a thrombus in the pulmonary system will result in decreased breath sounds and dyspnea. Pain in the residual limb is related not to a pulmonary embolus but to severed nerve endings in the residual limb. A pulmonary embolus will not interfere with arterial circulation to a distal portion of an extremity. Blanching of the affected extremity is associated with interference with arterial circulation to an extremity.

A client comes to the ambulatory surgery unit on the morning of an elective surgical procedure. The client reports shortness of breath, dizziness, and palpitations. The nurse observes profuse diaphoresis and is concerned that the client may be having either a panic attack or a myocardial infarction. Which assessments support the conclusion that the client may be experiencing a myocardial infarction? Select all that apply. 1. Anxiety 2. Chest pain 3. Irregular pulse 4. Fear of losing control 5. Feelings of depersonalization

1, 2, 3 Anxiety is associated with both myocardial infarctions and panic attacks. The overwhelming chest pain that usually accompanies a myocardial infarction, due to myocardial ischemia, precipitates a feeling of impending death. Most people who have panic attacks eventually recognize that they are not going to die as a result of the attack. Chest pain is associated with both myocardial infarctions and panic attacks. Chest pain is associated with a myocardial infarction because of myocardial ischemia. It is often described as "vice-like" or "crushing" in nature. The chest discomfort during a panic attack usually is not as severe as the pain associated with a myocardial infarction. Dysrhythmias often accompany a myocardial infarction because the functioning of the electrical pathways and cardiac muscles in the heart is impaired. Clients having a panic attack may have palpitations and tachycardia. Fear of losing control usually is not a characteristic associated with a myocardial infarction. Fear of losing control or going crazy is among the criteria of the DSM-IV-TR for the diagnosis of panic attacks. A feeling of depersonalization is not a characteristic associated with a myocardial infarction. Depersonalization (feeling detached from the self) or derealization (feelings of unreality) are among the criteria of the DSM-IV-TR for the diagnosis of panic attacks.

A client who has a hemoglobin of 6 gm/dL is refusing blood because of religious reasons. What is the most appropriate action by the nurse? 1. Call the chaplain to convince the client to receive the blood transfusion. 2. Discuss the case with coworkers. 3. Notify the primary health care provider of the client's refusal of blood products. 4. Explain to the client that they will die without the blood transfusion.

3 The nurse serves as an advocate for the client to uphold their wishes. Synthetic blood products are available but must be prescribed by the primary health care provider. Therefore, the primary health care provider needs to be notified of the client's refusal for blood so alternatives can be considered. . The chaplain's role is to offer support, not to convince the client to go against their beliefs. It is a HIPAA violation to discuss the case with coworkers unless they are involved in the care of the client. The nurse should not use threats or fear to coerce the client.

A client is admitted to the intensive care unit in pulmonary edema. What should the nurse expect when performing the admission assessment? 1. Weak rapid pulse 2. Decreased blood pressure 3. Radiating anterior chest pain 4. Crackles at the base of each lung

4 Crackles are the sound of air passing through fluid in the alveolar spaces. With pulmonary edema, fluid moves from the intravascular compartment into the alveoli. With hypervolemia, the pulse is bounding. The blood pressure is increased with hypervolemia. Radiating anterior chest pain will occur with angina or a myocardial infarction.

What should a nurse do to decrease or control the sensory and cognitive disturbances that can occur after a client has open heart surgery? 1. Restrict family visits. 2. Withhold analgesic medications. 3. Plan for maximum periods of rest. 4. Keep the room light on most of the time.

3 Sleep deprivation alone can cause these disturbances because of the interruption in rapid eye movement (REM) sleep. Lack of contact with significant others increases anxiety and feelings of isolation, which can lead to disturbances in rest. Pain limits or interrupts periods of sleep and rest. Analgesics should be administered as prescribed. Constant light increases cerebral arousal and limits sleep.

A nurse is collecting data from a client with varicose veins who is to have sclerotherapy. What should the nurse expect the client to report? 1. Feeling of heaviness in both legs. 2. Intermittent claudication of the legs. 3. Calf pain on dorsiflexion of the foot. 4. Hematomas of the lower extremities.

1 Impaired venous return causes increased pressure, with symptoms of fatigue and heaviness. Pain when walking relieved by rest (intermittent claudication) is a symptom related to hypoxia. Symptoms of hypoxia are related to impaired arterial, rather than venous, circulation. Calf pain on dorsiflexion of the foot is Homan sign, which is suggestive of thrombophlebitis. Ecchymoses may occur in some individuals, but bleeding into tissue is insufficient to cause hematomas.

A client admitted to the hospital for chest pain is diagnosed with angina. The nurse should teach the client that the most common characteristic of anginal pain is that it is: 1. Relieved by rest 2. Precipitated by light activity 3. Described as sharp or knifelike 4. Unaffected by the administration of vasodilators

1 Anginal pain commonly is relieved by immediate rest because rest decreases the cardiac workload. Angina usually is precipitated by exertion, emotion, or a heavy meal. Anginal pain usually is described as tightness, indigestion, or heaviness. Nitroglycerin dilates coronary arteries, which increases oxygen to the myocardium, decreasing pain.

A client calls out to all nursing staff members who pass by the door and asks them to do or get something. How can the nurse best manage this problem while meeting this client's needs? 1. Assign one staff member to approach the client regularly and interact with the client. 2. Close the door to the room so that the client cannot see the staff members as they pass by. 3. Inform the client that one staff member will come in frequently and check whether the client has any requests. 4. Arrange for a variety of staff members to take turns going into the room to see whether the client has any requests.

1 Assigning one staff member to approach the client regularly and interact with the client provides continuity and demonstrates to the client that the nursing staff is concerned; frequent contact should reduce the client's need to call the staff for reassurance. Closing the door to the room so that the client cannot see the staff members as they pass by may increase the client's anxiety and the need for contact with staff. Telling the client is not the same as doing it; the client may not believe that staff will come in frequently. Arranging for a variety of staff members to take turns going into the room to see whether the client has any requests will not facilitate the development of a therapeutic relationship with a staff member.

A client with a history of coronary artery disease is admitted with pneumonia. The health care provider prescribes atenolol (Tenormin). What should the nurse monitor to determine the therapeutic effect of atenolol? 1. Heart rate 2. Respirations 3. Temperature 4. Pulse oximetry

1 Atenolol, a beta blocker, slows the rate of Sinoatrial (SA) node discharge and AV node conduction, thus decreasing the heart rate; it prevents angina by decreasing the cardiac workload and myocardial oxygen consumption. Atenolol does not alter the respiratory rate, but its side effects may include bronchospasm and wheezing. Atenolol is not an antipyretic. Atenolol does not directly affect gas exchange in the lungs.

A client is diagnosed with esophageal varices and is admitted to the hospital. The health care provider prescribes a blood transfusion. What nursing actions should be taken? 1. Take the vital signs, verify the blood product with another nurse against the client's identification (ID) bracelet, and monitor the vital signs according to agency policy. 2. Because the vital signs were recorded during admission, hang the blood and monitor the client's vital signs every 15 minutes until the transfusion is absorbed. 3. Record the vital signs in accordance with facility policy and check the blood product against the client's ID bracelet in the presence of the nursing supervisor. 4. Take the vital signs after hanging the blood because the client is pale and moaning and is in critical condition; return in 15 minutes to monitor the vital signs.

1 Baseline vital signs should be obtained immediately before administering the blood product for future comparison purposes. Two licensed nurses should confirm the verifying data between the client and the blood product. The nurse should remain with and monitor the client's vital signs during the first 15 minutes of administration of the blood product and then follow the institution's protocol to monitor for a transfusion reaction or fluid overload. Vital signs must be taken immediately before the blood product infusion is begun for accurate future comparisons. It is not necessary for the licensed nurse verifying the data between the client and the blood product to be a supervisor. Blood should not be hung without following the appropriate protocol for ensuring accuracy of the blood product for the client; the nurse should remain with and monitor the client's vital signs during the first 15 minutes of administration of the blood product to monitor for a transfusion reaction or fluid overload.

A client is scheduled to be transferred from the coronary care unit to a progressive care unit. The client asks a nurse, "Are you sure I'm ready for this move?" What does the nurse conclude the client most likely is experiencing? 1. Fear 2. Depression 3. Dependency 4. Ambivalence

1 Fear of another myocardial infarct or sudden death is common when the client's environment is to be changed to one that appears less vigilant. Depression is exhibited by withdrawal, crying, anorexia, and apathy and usually becomes more evident after discharge from the hospital. Dependency usually is exhibited by an unwillingness to increase exercise or perform tasks. Ambivalence is exhibited by contrasting emotions; the client's statement does not demonstrate contrasting emotions.

A woman comes to the office of her health care provider, reporting shortness of breath and epigastric distress that is not relieved by antacids. To what question would a woman experiencing a myocardial infarction respond differently than a man? 1. "Do you have chest pain?" 2. "Are you feeling anxious?" 3. "Do you have any palpitations?" 4. "Are you feeling short of breath?"

1 Females may present with atypical symptoms of myocardial infarction, such as absence of chest pain, overwhelming fatigue, and indigestion. Anxiety, palpitations, and shortness of breath are common clinical manifestations in both males and females who are experiencing a myocardial infarction.

The client is diagnosed with peripheral arterial disease (PAD) and the nurse is discussing lifestyle modifications. Which of these is the most beneficial lifestyle modification the nurse should teach this client? 1. Stop smoking 2. Take an aspirin once daily 3. Start a walking program 4. Eat a low-fat, low-cholesterol diet

1 In clients with PAD, the goal is to promote vasodilation and prevent vasoconstriction. Complete abstinence from smoking or chewing tobacco products is considered the most effective method of preventing vasoconstriction. Smoking cessation should be encouraged. Although aspirin or other antiplatelets drugs are often prescribed for clients with PAD, it is not the primary means of risk reduction. Although a sedentary lifestyle is a contributing factor, it is not the primary risk factor for PAD. Although a high-fat, high-cholesterol diet is a contributing factor, it is not the primary risk factor for PAD.

What effect of anxiety makes it particularly important for the nurse to reduce the anxiety of a client with heart failure? 1. Increases the cardiac workload 2. Interferes with usual respirations 3. Produces an elevation in temperature 4. Decreases the amount of oxygen used

1 Irritability and restlessness associated with anxiety increase the metabolic rate, heart rate, and blood pressure; these complicate heart failure. Anxiety does not directly interfere with respirations; an increase in cardiac workload will increase respirations. Anxiety alone usually does not elevate the body temperature. Anxiety can cause an increase in the amount of oxygen used and leads to an increased respiratory rate.

A client with chronic bronchitis smokes one or two cigarettes a day and has not been performing the prescribed pulmonary physiotherapy exercises because they are too tiring. What is the best response by the nurse? 1. "Tell me about your typical day before the exercises were prescribed." 2. "Smoking is probably the cause of the severity of your disease at this time." 3. "Your being so sick is probably because of your smoking, and your choosing not to exercise." 4. "I can't make you stop doing what you are doing, and it's your choice to be sick or well."

1 More data are needed about activities of daily living to evaluate noncompliance before revising the care plan. "Smoking is probably the cause of the severity of your disease at this time," "Your being so sick is probably because of your smoking, and your choosing not to exercise," and "I can't make you stop doing what you are doing, and it's your choice to be sick or well" are nonproductive responses because it places blame for the illness on the client. Test-Taking Tip: Do not select answers that contain exceptions to the general rule, controversial material, or degrading responses.

The client's atrial and ventricular heart rates are equal at 88 beats/min. The PR interval is 0.14 seconds and the QRS width is 0.10 seconds. Rhythm is regular with normal P waves and QRS complexes. The client's rhythm is: 1. Normal sinus rhythm (NSR). 2. Sinus tachycardia. 3. Sinus bradycardia. 4. Sinus arrhythmia.

1 NSR reflects normal conduction of the sinus impulse through the atria and ventricles. Atrial and ventricular rates are the same and range from 60 to 100 beats per minute. Rhythm is regular or essentially regular. PR interval is 0.12 to 0.20 seconds. QRS interval is 0.06 to 0.10 seconds. P and QRS waves are consistent in shape. Sinus tachycardia results when the sinoatrial (SA) node fires faster than 100 beats per minute. Bradycardia is defined as a heart rate less than 60 beats per minute. Sinus arrhythmia is a cyclical change in heart rate that is associated with respiration. The heart rate slightly increases during inspiration and slightly slows during exhalation because of changes in vagal tone.

A client who just returned from surgery reports shortness of breath and chest pain. Which should the nurse initially administer? 1. Supplemental oxygen 2. Intravenous morphine 3. Endotracheal intubation 4. Sublingual nitroglycerin

1 Oxygen supports vital centers of the body while the cause of the problem is investigated. Although an intravenous morphine may be done eventually if the client is experiencing a myocardial infarction, it is not the initial action and requires a prescription. Endotracheal intubation is not implemented by a nurse. Later, endotracheal intubation may be necessary if the client experiences respiratory failure or obstruction. Although a sublingual nitroglycerin may be done eventually if the client is experiencing angina, it is not an initial action and requires a prescription.

A client who has peripheral arterial disease of the lower extremities tells the nurse, "I walk so slowly that no one wants to walk with me." What is the best response by the nurse? 1. "A vascular rehabilitation program may help you." 2. "You should be sitting with your feet elevated, not walking." 3. "Try again tomorrow because maybe you will have a better day." 4. "They are not good friends if they are not willing to walk with you."

1 Peripheral vascular rehabilitation includes exercise and walking programs that encourage new growth of vessels around the obstructed artery; this may improve peripheral perfusion and the ability to walk; eventually, walking with friends may be introduced into the walking program. Inactivity is contraindicated; elevation of the legs diminishes peripheral arterial circulation. The response "Try again tomorrow because maybe you will have a better day" provides false reassurance. The response "They are not good friends if they are not willing to walk with you" is an opinion that should be avoided; it does not focus on the client's need to improve walking ability.

A nurse reviews the plan of care for a client who is recovering from the acute phase of left ventricular failure. The nurse expects which dietary restriction to be included on the plan? 1. Sodium 2. Calcium 3. Potassium 4. Magnesium

1 Restriction of sodium reduces the amount of water retention, thus reducing cardiac workload. Calcium is restricted in individuals who develop renal calculi with a calcium phosphate base. Potassium is not restricted, especially if a diuretic and cardiac glycoside are prescribed because these drugs facilitate the loss of potassium in the urine. Magnesium is not restricted.

After surgery a client's fever does not respond to antipyretics. The health care provider prescribes that the client be placed on a hypothermia blanket. A response to hypothermia therapy that the nurse should prevent is: 1. Shivering 2. Vomiting 3. Dehydration 4. Hypotension

1 Shivering should be prevented; peripheral vasoconstriction increases the temperature, the circulatory rate, and oxygen consumption. Vomiting is not a response to hypothermia therapy. Dehydration is not a response to hypothermia therapy; presence of a fever can cause dehydration if oral or parenteral fluid intake is inadequate to maintain fluid balance. Hypotension is not a response to hypothermia therapy; hypotension can occur with dehydration if oral or parenteral fluid intake is inadequate to maintain fluid balance.

How should the nurse monitor for the complication of subcutaneous emphysema after the insertion of chest tubes? 1. Palpate around the tube insertion sites for crepitus. 2. Auscultate the breath sounds for crackles and rhonchi. 3. Observe the client for the presence of a barrel-shaped chest. 4. Compare the length of inspiration with the length of expiration.

1 Subcutaneous emphysema occurs when air leaks from the intrapleural space through the thoracotomy or around the chest tubes into the soft tissue; crepitus is the crackling sound heard when tissues containing gas are palpated. Crackles and rhonchi are unrelated to crepitus. They occur within the lung; subcutaneous emphysema occurs in the soft tissues. Observing the client for the presence of a barrel-shaped chest is related to prolonged trapping of air in the alveoli associated with emphysema, a chronic obstructive pulmonary disease. Comparing the length of inspiration with the length of expiration is unrelated to subcutaneous emphysema, which involves gas in the soft tissues from a pleural leak.

A 2 g sodium diet is prescribed for a client with stage 2 hypertension, and the nurse teaches the client the rationale for this diet. The client reports distaste for the food. The primary nurse hears the client request that the family "bring in a ham and cheese sandwich and fries." What is the most effective nursing intervention? 1. Discuss the diet with the client and family. 2. Tell the client why salty foods should not be eaten. 3. Explain the dietary restriction to the client's visitors. 4. Ask the dietitian to teach the client and family about sodium restrictions.

1 The client and significant family members should be included in dietary teaching; families provide support that promotes adherence. The client already has received information about why salty foods should not be eaten. Explaining the dietary restriction to the client's visitors could violate confidentiality. The client should be involved in his or her own care; the client ultimately will assume the responsibility. The dietitian is a resource person who can give specific, practical information about diet and food preparation once there is a basic understanding of the reasons for the diet.

A client diagnosed with asthma has received a prescription for an inhaler. The nurse teaches the client how to determine when the inhaler is empty, instructing the client to: 1. Count the number of doses taken 2. Taste the medication when sprayed into the air 3. Shake the canister 4. Place the canister in water to see if it floats

1 The only way to determine if the canister is empty is to count the number of doses taken. The client is tracking the number of daily doses. It is wasteful to spray medication into the air; tasting it from the air is not an effective method of determining if the canister is empty. Shaking the canister is not effective; even if there is no more medication, some propellant may be left. It is futile to place the canister in water; the flotation test is ineffective.

For which expected response should the nurse monitor a client after a cardiac catheterization? 1. Marked increase in the volume of urine output 2. Decrease in blood pressure of 25% from the precatheterization blood pressure 3. Complaints of heart pounding with mild chest discomfort 4. Respiratory distress with an increase in respiratory rate more than 24 respirations per minute

1 There is increased urinary output as a result of the diuretic effect of the contrast medium. A decrease of 10% to 20% is expected because of the diuretic effect of the contrast medium; a decrease greater than 20% may be pathologic. Although heart pounding with mild chest discomfort may occur during the procedure because of trauma to the conduction system, it usually does not continue after the procedure. Respiratory distress may be an indication of a pulmonary embolus from a venous clot and should be reported immediately.

A hospitalized client puts the call light on and reports a sudden onset of chest pain that feels like a pressure or weight on the chest. The client also states, "I feel nauseated and very weak." What action should the nurse take? 1. Call the rapid response team 2. Perform a nutritional assessment 2. Discuss possible sources of stress with the client 4. Provide reassurance while helping the client to focus on pleasant topics

1 These are classic symptoms of a myocardial infarction; further medical evaluation and intervention are needed immediately. Performing a nutritional assessment presumes a dietary problem when a more serious situation may exist. Discussing possible sources of stress with the client considers only an emotional source of the reported symptoms and ignores a potential medical emergency. Providing reassurance while helping the client to focus on pleasant topics provides false reassurance and ignores a potential medical emergency.

What clinical indicator is the nurse most likely to identify when completing a history and physical assessment of a client with complete heart block? 1. Syncope 2. Headache 3. Tachycardia 4. Hemiparesis

1 With complete atrioventricular block, the ventricles take over the pacemaker function in the heart but at a much slower rate than that of the sinoatrial (SA) node. As a result, there is decreased cerebral circulation, causing syncope. Headache is not related to heart block. The heart rate usually is slow because the ventricular rhythm is not initiated by the SA node. Hemiparesis is not related to heart block unless decreased cerebral perfusion causes a brain attack.

A child has been diagnosed with hemophilia type A after experiencing excessive bleeding from a minor trauma. The parents ask what are the symptoms of bleeding for which they should be looking in the future. What are the signs for which to look? Select all that apply. 1. Epistaxis 2. Hematuria 3. Hemarthrosis 4. Easy bruising 5. Frequent fevers 6. Fast clotting of injuries 7. Dark colored tarry stools

1, 2, 3, 4, 7 Epistaxis, also known as nosebleeds, is a common symptom of a lack of clotting factor. Hematuria (blood in the urine) may be grossly apparent. The child may experience joint pain and deformities from bleeding into joints. Excessive bruising will occur from bleeding into tissue with seemingly minor injuries. Dark colored tarry consistency stools are indicative of gastrointestinal bleeding. Frequent fevers are not associated with hemophilia. Prolonged clotting times occur with this condition.

How should a nurse prepare an intravenous piggyback (IVPB) medication for administration to a client receiving an IV infusion? Select all that apply. 1. Wear clean gloves to check the IV site. 2. Rotate the bag after adding the medication to mix. 3. Use 100 mL of fluid to mix the medication. 4. Flush the IV insertion site with 2 mL saline. 5. Place the IVPB at a lower level than the existing IV. 6. Use a sterile technique when preparing the medication.

1, 2, 6 Clean gloves should be worn to check the IV site because there is a risk of coming into contact with the client's blood. Ensuring that the medication is mixed is important. Rotating the bag is one way, although there are others. Because IV solutions enter the body's internal environment, all solutions and medications using this route must be sterile to prevent the introduction of microbes. The amount and type of solution depend on the medication. The insertion site does not have to be flushed with an infusing IV. The IVPB should be hung higher, not lower, than the existing bag.

A client is informed that he has developed a health care-associated upper respiratory tract infection and asks the nurse what this means. How should the nurse reply? 1. "You developed an infection that requires antibiotics." 2. "You acquired the infection after being admitted to the hospital." 3. "An infection you had before beginning treatment has flared up." 4. "This is a highly contagious infection requiring complete isolation."

2 A health care-associated infection (formerly called nosocomial infection) is contracted during the course of receiving treatment. Although developing an infection that requires antibiotics may occur, this response does not explain a health care-associated infection. A preexisting infection is unrelated to a health care-associated infection. The need for precautions relates to the type of infection, not to the situation in which it was acquired.

On the first day after a right pneumonectomy, a client suddenly sits straight up in bed. The client's respirations are labored, and a crowing sound is audible. The client's skin is pale, cool, and moist. The priority nursing intervention is: 1. Notify the primary healthcare provider 2. Auscultate the left lung 3. Inspect the incision for bleeding 4. Check the chest tube for patency

2 A mediastinal shift with airway obstruction may occur because pressure builds up on the operative side, causing the trachea to deviate toward the nonoperative side; assessment of the airway takes priority. Notifying the health care provider eventually will be done; further assessment is the priority. Inspecting the incision for bleeding is not the priority; further assessment is the priority. There is no need for a chest tube when a pneumonectomy is performed.

A nurse is caring for a client with an infection caused by group A beta-hemolytic streptococci. The nurse should assess this client for responses associated with which illness? 1. Hepatitis A 2. Rheumatic fever 3. Spinal meningitis 4. Rheumatoid arthritis

2 Antibodies produced against group A beta-hemolytic streptococci sometimes interact with antigens in the heart's valves, causing damage and symptoms of rheumatic heart disease; early recognition and treatment of streptococcal infections have limited the occurrence of rheumatic heart disease. Hepatitis A, an inflammation of the liver, is caused by the hepatitis A virus (HAV), not by bacteria. The most common causes of meningitis, an infection of the membranes surrounding the brain and spinal cord, include Streptococcus pneumoniae, Neisseria meningitides, and Haemophilus influenzae. Rheumatoid arthritis is thought to be an autoimmune disorder; it is not caused by microorganisms.

When assessing the breath sounds of a client with chronic obstructive pulmonary disease (COPD), the nurse hears coarse rhonchi. They are described best as: 1. Snorting sounds during the inspiratory phase 2. Moist rumbling sounds that clear after coughing 3. Musical sounds more pronounced during expiration 4. Crackling inspiratory sounds unchanged with coughing

2 Coarse rhonchi, particularly on expiration, indicate partial airway obstruction because of bronchiolar alterations associated with COPD. Snorting sounds are made in the nose. Wheezes are musical sounds usually heard during expiration; they are caused by rapid vibration of bronchial walls. Crackling sounds heard on inspiration that are unchanged by coughing are known as fine crackles; they result when air passes through alveoli that partially are filled with fluid.

A client with supraventricular tachycardia (SVT) is being treated with diltiazem hydrochloride (Cardizem). What assessment indicates to the nurse that the diltiazem hydrochloride is effective? 1. Blood pressure of 90/60 mm Hg 2. Heart rate of 110 beats per minute 3. No longer complaining of heart palpations 4. Increased urine output

2 Diltiazem hydrochloride's purpose is to slow the heart rate down. SVT has a heart rate of 150 to 250 beats per minute. A heart rate of 110 indicates that the diltiazem hydrochloride is having the desired effect. Hypotension is a side effect of diltiazem hydrochloride, not a desired effect. Heart palpations are experienced by some with various dysrhythmias. A decreased sensation of heart palpations is a positive finding but is not present in all clients. Increased urine output may occur over a period of time because of the increased ventricular filling time, but would not occur until after the heart rate had stabilized.

A nurse manager whose leadership style is authoritarian can be described as:

2 Leaders who exert strong control, are self-righteous, and expect others to follow them are known as authoritarian leaders. Leaders who are permissive and provide a minimum of leadership participation are known as laissez-faire leaders. Democratic leaders believe in collaboration and encourage others to contribute to the work effort. There is no leadership style called contributive. Democratic leaders believe in encouraging others to contribute to the work effort.

A client is in cardiogenic shock. What explanation of cardiogenic shock should the nurse include when responding to a family member's questions about the condition? 1. An irreversible phenomenon 2. A failure of the circulatory pump 3. Usually a fleeting reaction to tissue injury 4. Generally caused by decreased blood volume

2 Shock may have different etiologies (e.g., hypovolemic, cardiogenic, septic, anaphylactic) but always involves a drop in blood pressure and failure of the peripheral circulation because of sympathetic nervous system involvement. In cardiogenic shock, the failure of peripheral circulation is caused by the ineffective pumping action of the heart. Shock can be reversed by the administration of fluids, plasma expanders, and vasoconstrictors. It may be a reaction to tissue injury, but there are many different etiologies (e.g., hypovolemia, sepsis, anaphylaxis); it is not fleeting. Hypovolemia is only one cause.

Hydrochlorothiazide (HCTZ), a thiazide diuretic, has been prescribed for a client with hypertension. The client, who does not know about HCTZ, asks the nurse why furosemide (Lasix) was not prescribed. How should the nurse respond? 1. "HCTZ does not cause dizziness." 2. "HCTZ has fewer side effects." 3. "HCTZ does not cause dehydration." 4. "HCTZ is only taken when needed."

2 Side effects from thiazides generally are minor and rarely result in discontinuation of therapy. Dizziness is a side effect of all diuretics. There is a potential for dehydration with all diuretics. All diuretic medications are taken regularly as directed.

When a client develops ventricular fibrillation in a coronary care unit, what is the responsibility of the first person reaching the client? 1. Administer oxygen. 2. Initiate defibrillation. 3. Initiate cardiopulmonary resuscitation (CPR). 4. Administer sodium bicarbonate intravenously.

2 Ventricular fibrillation is a lethal dysrhythmia and, once identified, must be terminated immediately by precordial shock (defibrillation) so the sinus node can act again as the heart's pacemaker. This usually is a standing prescription in a coronary care unit. Oxygen is administered to correct hypoxia; it does not take priority over defibrillation. CPR is instituted only when defibrillation fails to terminate the dysrhythmia. Bicarbonate is administered to correct acidosis; it does not take priority over defibrillation.

When an older client with heart failure is transferred from the emergency department to the medical service, what should the nurse on the unit do first? 1. Interview the client for a health history. 2. Assess the client's heart and lung sounds. 3. Monitor the client's pulse and temperature. 4. Obtain the client's blood specimen for electrolytes.

2 With heart failure the left ventricle is not functioning effectively, which is evidenced by an increased heart rate and crackles associated with pulmonary edema. Interviewing the client for a health history is done after vital signs and breath sounds are obtained and the client is stabilized. Although an infection would complicate heart failure, there are no signs that indicate this client has an infection. Obtaining the client's blood specimen for electrolytes is inappropriate for immediate monitoring; it should be done after vital signs and clinical assessments have been completed.

When assessing a client with pleural effusion, the nurse expects to identify: 1. Moist crackles at the posterior of the lungs 2. Deviation of the trachea toward the involved side 3. Reduced or absent breath sounds at the base of the lung 4. Increased resonance with percussion of the involved area

3 Compression of the lung by fluid that accumulates at the base of the lungs reduces lung expansion and air exchange. There is no fluid in the alveoli, so no cracklesare produced. If there is tracheal deviation, it is away from the involved side. Dullness is produced on percussion of the involved area.

A nurse receives a call from the emergency department about a client with tuberculosis (TB) who will be admitted to the medical unit. What precautions should the nurse take? 1. Put on a gown when entering the room. 2. Place the client with another client who has TB. 3. Wear a particulate respirator when caring for the client. 4. Don a surgical mask with a face shield when entering the room.

3 A high-particulate filtration mask that meets Center for Disease Control (CDC) performance criteria for a tuberculosis respirator must be worn to protect health care providers from exposure to the Mycobacterium tuberculosis organism. Airborne transmission-based precautions do not require a gown unless contact with respiratory secretions is anticipated. The client should be placed in a private room with negative pressure and multiple full air exchanges per hour vented to the outside environment. A surgical mask with a face shield is inadequate to prevent transmission of the tuberculosis microorganism.

The nurse is caring for a client that is on a low carbohydrate diet. With this diet, there is decreased glucose available for energy, and fat is metabolized for energy resulting in an increased production of which substance in the urine? 1. Protein 2. Glucose 3. Ketones 4. Uric acid

3 As a result of fat metabolism, ketone bodies are formed and the kidneys attempt to decrease the excess by filtration and excretion. Excessive ketones in the blood can cause metabolic acidosis. A low carbohydrate diet does not cause increased protein, glucose, or uric acid in the urine.

A client with a distal femoral shaft fracture is at risk for developing a fat embolus. The nurse considers that a distinguishing sign that is unique to a fat embolus is: 1. Oliguria 2. Dyspnea 3. Petechiae 4. Confusion

3 At the time of a fracture or orthopedic surgery, fat globules may move from the bone marrow into the bloodstream. Also, elevated catecholamines cause mobilization of fatty acids and the development of fat globules. In addition to obstructing vessels in the lung, brain, and kidneys with systemic embolization of small vessels from fat globules, petechiae are noted in the buccal membranes, conjunctival sacs, hard palate, chest, and anterior axillary folds; these adaptations only occur with a fat embolism. Oliguria, dyspnea, and confusion are signs of an embolus, but are not specific to a fat embolus.

What is the most effective way for the nurse to loosen respiratory secretions for a client with an endotracheal tube? 1. Increase oral fluid intake. 2. Provide chest physiotherapy. 3. Humidify the prescribed oxygen. 4. Instill a saturated solution of potassium iodide.

3 Because the client has an endotracheal tube in place, secretions can be loosened by administration of humidified oxygen and by frequent turning. A client with an endotracheal tube in place is not permitted fluids by mouth. Providing chest physiotherapy is too vigorous for a client with an endotracheal tube. Potassium is never instilled into the lungs.

A 40-year-old client states, "I feel like my heart is jumping out of my chest, and it is skipping beats." The client passes a thallium stress test; however, the health care provider identifies one premature ventricular complex (PVC) and several premature atrial complexes (PACs) on the 24-hour follow-up Holter monitor. Which question is most important for the nurse to ask the client? 1. "Do you have small children at home?" 2. "Are you eating foods high in vitamins?" 3. "How much caffeine do you consume each day?" 4. "How many glasses of water do you drink per day?"

3 Caffeine is a stimulant that causes the heart to become irritable; it can result in tachycardia and atrial dysrhythmias. Vitamins are unrelated to an irregular heart rhythm. Small children and water consumption are unrelated to the client's physical problem.

An older client is apprehensive about being hospitalized. The nurse realizes that one of the stresses of hospitalization is the unfamiliarity of the environment and activity. How can the nurse best limit the client's stress? 1. Use the client's first name. 2. Visit with the client frequently. 3. Explain what the client can expect. 4. Listen to what the client has to say.

3 Explaining procedures and routines should decrease the client's anxiety about the unknown. The nurse should not confuse roles of professional and friend; the client should be called by an appropriate title (Mr., Miss, Ms., Mrs., etc.) unless the client requests otherwise. The nurse should not confuse the role of professional with that of being a friend; "visiting" has a social connotation. Although listening to the client is therapeutic, this does not change the fact that the hospital environment is strange to the client and the client needs information.

An emergency department nurse is admitting a client after an automobile collision. The health care provider estimates that the client has lost about 15% to 20% of blood volume. Which assessment finding should the nurse expect this client to exhibit? 1. Urine output of 50 mL/hr 2. Blood pressure of 150/90 mm Hg 3. Apical heart rate of 142 beats/min 4. Respiratory rate of 16 breaths/min

3 In hypovolemic shock, tachycardia is a compensatory mechanism in an attempt to increase blood flow to body organs. Urine output would fall to less than 30 mL/hr because a decreased blood volume causes a decreased glomerular filtration rate. The blood pressure is decreased because of the decreased blood volume. Respiratory rate of 16 breaths/min is within the accepted range of 12 to 20 breaths/min; the respiratory rate is rapid with hypovolemic shock.

A client is admitted with metabolic acidosis. The nurse considers that two body systems interact with the bicarbonate buffer system to preserve healthy body fluid pH. What two body systems should the nurse assess for compensatory changes? 1. Skeletal and nervous 2. Circulatory and urinary 3. Respiratory and urinary 4. Muscular and endocrine

3 Increased respirations blow off carbon dioxide (CO2), which decreases the hydrogen ion concentration and the pH increases (less acidity). Decreased respirations result in CO2 buildup, which increases hydrogen ion concentration and the pH falls (more acidity). The kidneys either conserve or excrete bicarbonate and hydrogen ions, which helps to adjust the body's pH. The buffering capacity of the renal system is greater than that of the pulmonary system, but the pulmonary system is quicker to respond. Skeletal and nervous systems do not maintain the pH, nor do muscular and endocrine systems. Although the circulatory system carries fluids and electrolytes to the kidneys, it does not interact with the urinary system to regulate plasma pH.

The nurse should place the client in which position to obtain the most accurate reading of jugular vein distention? 1. Upright at 90 degrees 2. Supine position 3. Raised to 30 degrees 4. Raised to 10 degrees

3 Jugular vein pressure is measured with a centimeter ruler to obtain the vertical distance between the sternal angle and the point of highest pulsation. This procedure is most accurate when the head of the bed is elevated between 30 and 45 degrees. The internal and external jugular veins should be inspected while the client is gradually elevated from a supine position to an upright 30-45 degrees. Jugular vein distention cannot accurately be assessed if the client is supine, at 90 degrees or 10 degrees.

A client is to have mafenide (Sulfamylon) cream applied to burned areas. For which serious side effect of mafenide therapy should the nurse monitor this client? 1. Curling ulcer 2. Renal shutdown 3. Metabolic acidosis 4. Hemolysis of red blood cells

3 Mafenide interferes with the kidneys' role in hydrogen ion excretion, resulting in metabolic acidosis. Curling ulcer, renal shutdown, and hemolysis of red blood cells are not adverse effects of the drugs.

A client that is postoperative hip replacement is receiving morphine by patient-controlled analgesia and has a respiratory rate of 6 breaths/min. What intervention should the nurse anticipate? 1. Nasotracheal suction 2. Mechanical ventilation 3. Naloxone administration 4. Cardiopulmonary resuscitation

3 Naloxone is an opioid (narcotic) antagonist and will reverse respiratory depression caused by opioids. Nasotracheal suction, mechanical ventilation, and cardiopulmonary resuscitation are not needed; naloxone will correct the respiratory depression.

An 85-year-old client with a history of congestive heart failure on daily weights has a 2-pound weight gain and pitting edema in lower extremities bilaterally, indicative of fluid retention. The most appropriate response is: 1. Check the record to ascertain the code status 2. Notify the primary health care provider of the change 3. Perform a head-to-toe assessment, including vital signs 4. Continue to monitor daily weights and edema and document findings

3 Performing a head-to-toe assessment, including vital signs, would indicate symptoms, such as jugular distention with right sided heart failure, or pulmonary issues associated with left-sided heart failure. Checking the record for code status is not a priority and should have been established and known on an elderly client. Notifying the health care provider of the weight change and edema without full vital signs and assessment would not be helpful as this information will be needed. Continuing to monitor daily weights without an assessment may miss worsening symptoms.

The treatment regimen for a female diagnosed with Hodgkin disease, stage III, will start with nodal irradiation. Because the client and her husband have been trying to conceive a child, the client becomes visibly anxious when she learns that the radiation therapy includes the pelvic nodal area. When questioned about this, the nurse should refer the client to the primary health care provider because: 1. Radiation used is not radical enough to destroy ovarian function 2. Intermittent radiation to the area does not cause permanent sterilization 3. Reproductive ability may be preserved through a variety of interventions 4. Ovarian function will be destroyed temporarily but will return in about six months

3 Reproductive ability may be preserved through shielding the ovaries or harvesting ova. Radiation can influence or destroy ovarian functioning. Sterilization can occur. Women in the childbearing years should be informed of all options available to preserve ovarian function. Once ova are destroyed, they cannot regenerate.

A client is admitted to the hospital with a long history of hypertension. The nurse should assess the client for which complication? 1. Cataracts 2. Esophagitis 3. Kidney failure 4. Diabetes mellitus

3 Some renal impairment usually is present even with mild hypertension and is attributed to the ischemia resulting from narrowed renal blood vessels and increased intravascular pressure; decreased blood flow causes atrophy of renal structures, such as tubules, glomeruli, and nephrons, leading to kidney failure. Retinopathy, resulting in blurred vision, retinal hemorrhage, and blindness, occurs with a long history of hypertension because of increased intravascular pressure, not cataracts. Esophagitis is caused by esophageal reflux disease, not a long history of hypertension. Hypertension does not cause diabetes mellitus; however, chronic elevations of serum glucose accelerate atherosclerosis, resulting in the development of hypertension.

A primary nurse receives prescriptions for a newly admitted client and has difficulty reading the health care provider's writing. Who should the nurse ask for clarification of this prescription? 1. Nurse practitioner 2. House health care provider that is on-call 3. Health care provider who wrote the prescription 4. Nurse manager familiar with the health care provider's writing

3 The health care provider who wrote the prescription should be called for clarification. The nurse is liable and responsible if the prescription is misinterpreted. Only the health care provider who wrote an undecipherable prescription can correctly clarify the prescription.

A client's arterial blood gas report indicates that pH is 7.25, PCO2 is 35 mm Hg, and HCO3 is 20 mEq/L. Which client should the nurse consider is most likely to exhibit these results? 1. 54-year-old with vomiting 2. 17-year-old with panic attacks 3. 24-year-old with diabetic ketoacidosis 4. 65-year-old with advanced emphysema

3 The low pH and bicarbonate levels are consistent with metabolic acidosis, which can be caused by excess ketones, a result of diabetic ketoacidosis. A 54-year-old with vomiting most likely will experience metabolic alkalosis from loss of gastric hydrochloric acid. A 17-year-old with panic attacks most likely will experience metabolic alkalosis from hyperventilation. A 65-year-old with advanced emphysema most likely will experience respiratory acidosis.

A client is admitted to the emergency department with a possible myocardial infarction. Three hours after admission, the client experiences a new onset of severe chest pain. The client is diaphoretic with a pulse rate of 110 beats per minute. The nurse should immediately: 1. Increase the oxygen amount 2. Obtain the blood pressure and an electrocardiogram (ECG) 3. Administer the prescribed morphine and notify the primary health care provider 4. Administer the prescribed nitroglycerin tablet until the pain subsides

3 The original myocardial infarction may be extending; the client's adaptations require immediate medical intervention and relief of pain. Increasing the oxygen flow should be done after notification of the practitioner; notification and pain relief are the priorities. The blood pressure and an ECG will provide additional data but will cause a delay in notification of the health care provider and provision of necessary medical intervention. Nitroglycerin does not relieve the pain associated with a myocardial infarction.

Which nursing action is most important when caring for a client after a cardiac catheterization? 1. Provide for rest. 2. Administer oxygen. 3. Check a pulse distal to the insertion site. 4. Assess the electrocardiogram every 15 minutes.

3 The pulse should be assessed because the trauma at the insertion site may interfere with blood flow distal to the site. There is also danger of bleeding. Rest is not a priority, although the extremity in which the catheter was inserted usually is immobilized for a period of time to prevent bleeding at the insertion site. Whether to administer oxygen is determined on an individual basis; it is not routine. The client will be on a cardiac monitor, which will allow for continuous monitoring.

A client demonstrates signs and symptoms of a transfusion reaction. The nurse immediately stops the infusion and next: 1. Obtains blood pressure in both arms 2. Sends a urine specimen to the laboratory 3. Hangs a bag of normal saline with new tubing 4. Monitors the intake and output every 15 minutes

3 The tubing must be replaced to avoid infusing the blood left in the original tubing; the normal saline infusion will maintain an open line for any further intravenous (IV) treatment. All vital signs should be taken eventually; blood pressure may be taken on either arm, not necessarily both. A urine sample is collected after the blood transfusion is stopped, the tubing replaced, and a bag of normal saline hung. The specimen will be analyzed to determine kidney function. Although the intake, and especially the output, should be monitored to assess kidney function, this is not the priority.

The family of an older adult who is aphasic reports to the nurse manager that the primary nurse failed to obtain a signed consent before inserting an indwelling catheter to measure hourly output. What should the nurse manager consider before responding? 1. Procedures for a client's benefit do not require a signed consent. 2. Clients who are aphasic are incapable of signing an informed consent. 3. A separate signed informed consent for routine treatments is unnecessary. 4. A specific intervention without a client's signed consent is an invasion of rights.

3 This is considered a routine procedure to meet basic physiologic needs and is covered by a consent signed at the time of admission. The need for consent is not negated because the procedure is beneficial. This treatment does not require special consent.

A nurse in the postanesthesia care unit is caring for a client who received a general anesthetic. Which finding should the nurse report to the health care provider? 1. Client pushes the airway out. 2. Client has snoring respirations. 3. Respirations of 16 breaths/min are shallow. 4. Systolic blood pressure drops from 130 to 90 mm Hg.

4 A drop in blood pressure, rapid pulse rate, cold, clammy skin, and oliguria are signs of decreased blood volume and shock, which if not treated promptly can lead to death. The client pushing the airway out is an expected response; the client will push out the airway as the effects of anesthesia subside. Shallow respirations of 16 breaths/min and systolic blood pressure dropping to 130 to 190 mm Hg are common responses to depressant effects of anesthesia.

A client with a history of heart disease has been receiving a calcium channel blocker and morphine sulfate for pain from abdominal surgery. When getting the client out of bed, the nurse first should have the client sit on the edge of the bed with feet on the floor. What untoward client response can be prevented by this nursing action? 1. Abdominal pain 2. Respiratory distress 3. Sudden hemorrhage 4. Postural hypotension

4 After administration of certain antihypertensives or opioids, a client's neurocirculatory reflexes may have some difficulty adjusting to the force of gravity when an upright position is assumed. Postural or orthostatic hypotension occurs, and blood supply to the brain is temporarily decreased. Abdominal pain, respiratory distress, and sudden hemorrhage will not be prevented by the intervention described.

A client reports having a bad cold and chest pain that worsens when the client takes deep breaths. Where should the nurse place the stethoscope to determine the presence of a pleural friction rub? Choose the appropriate location marked on the image. 1. trachea 2. between the scapulae 3. where vesicular breath sounds are heard 4. lower-lateral chest

4 Answer D is the lower-lateral chest, which is the area of greatest thoracic excursion. With visceral and parietal pleural inflammation (pleurisy), a low-pitched, coarse, grating sound is heard when the client breathes, particularly when approaching the height of inspiration. Bronchial breath sounds are heard over the trachea and at the nape of the neck on either side of the vertebrae. Bronchial sounds are loud, high pitched, and hollow, with a short inspiratory phase and long expiratory phase. Bronchovesicular breath sounds are heard on either side of the sternum or between the scapulae; bronchovesicular sounds have a moderate volume and medium pitch, with equal inspiratory and expiratory phases. C is the area where vesicular breath sounds are heard. Vesicular sounds are soft and low pitched, with a long inspiratory phase and a short expiratory phase; they are heard over most lung fields.

A client with cancer of the tonsils and enlarged lymph glands in the neck is receiving chemotherapy after surgery. Which laboratory report is most important for the nurse to monitor when considering the effects of chemotherapy? 1. Platelets 2. Hemoglobin level 3. Red blood cell count 4. White blood cell count

4 Antineoplastic drugs depress bone marrow, which results in leukopenia; the client must be protected from infection, which may cause death. Platelets may decrease rapidly, but complications may be limited by infusions of platelets. Although the hemoglobin level diminishes, a transfusion with packed red blood cells (RBCs) will alleviate the anemia. RBCs diminish slowly and may be replaced with a transfusion of packed red blood cells.

During a client's routine physical examination, an abdominal aortic aneurysm is diagnosed. The client is admitted to the hospital immediately, and surgery is scheduled for the next morning. Which clinical finding should the nurse expect when completing the admission assessment? 1. Signs of shock 2. Visible peristaltic waves 3. Radiating abdominal pain 4. Pulsating abdominal mass

4 As the heart contracts, an expanding midline mass can be palpated to the left of the umbilicus. Signs of shock are not definitive for an abdominal aortic aneurysm unless the aneurysm ruptures. Visible peristaltic waves are associated with an intestinal obstruction. Radiating abdominal pain is not definitive for an abdominal aortic aneurysm.

A nurse is caring for a client who had radical neck surgery. For which complication associated with this surgery should the nurse assess this client? 1. Pulmonary edema 2. Cardiogenic shock 3. Atrophy of chest muscles 4. Rupture of the carotid artery

4 Because of the proximity of the carotid artery to the surgical area and the possibility that age or the disease process has weakened the carotid artery, the client should be monitored for signs of hemorrhage related to carotid rupture. Pulmonary edema is related to cardiac decompensation, which is not an expected complication of radical neck surgery. Cardiogenic shock is related to cardiac decompensation, which is not an expected complication of radical neck surgery. With a radical neck dissection the trapezius muscle, not chest muscles, may atrophy.

A client with impaired peripheral pulses and signs of chronic hypoxia in a lower extremity is scheduled for a femoral angiogram. What would be appropriate for the nurse to include in the postprocedure plan of care? 1. Elevate the foot of the bed 2. Perform urinary catheter care every 12 hours 3. Place in the high-Fowler position 4. Perform a neurovascular assessment every two hours

4 Because of the trauma associated with the insertion of the catheter during the procedure, the involved extremity should be assessed for sensation, motor ability, and arterial perfusion; hemorrhage or an arterial embolus can occur. The client has an arterial problem, and perfusion is promoted by keeping the legs at the level of or lower than the heart. A general anesthetic is not used; therefore, voiding is not a concern. Keeping the client in the high-Fowler position is unsafe; this position increases pressure in the groin area, which can dislodge the clot at the catheter insertion site, resulting in bleeding. It also impedes arterial perfusion and venous return.

A nurse has difficulty palpating the pedal pulse of a client with venous insufficiency. What action should the nurse take next? 1. Count the pulse at another site. 2. Notify the health care provider. 3. Lower the legs to increase blood flow. 4. Verify the pulse by using a Doppler.

4 Clients with venous insufficiency often have edema, which may make palpation of an arterial pulse difficult. A Doppler uses sound waves so that the pulse can be heard. The quality of the pedal pulse, not the rate, is assessed to determine the adequacy of peripheral arterial circulation; the most distal site is preferred. The nurse must make other assessments of circulation before notifying the health care provider. Lowering the legs will increase edema and make palpation of pulses more difficult.

A client who had a myocardial infarction requests assistance to have a bowel movement. The nurse should: 1. Place the client on a bedpan 2. Help the client into the bathroom 3. Roll the client onto a fracture pan 4. Assist the client to a bedside commode

4 Defecation in the sitting position on a bedside commode uses less energy than walking to the bathroom or getting on and off a bedpan. Defecation is difficult on a bedpan and may cause straining and an increase in oxygen demands. Walking to the bathroom uses more energy than using a bedside commode. Although the use of a fracture pan takes less energy than using a regular bedpan, it takes more energy than using a commode.

A client is admitted to the critical care unit after receiving multiple injuries in a motorcycle accident. Twelve hours later the client reports increased abdominal pain in the left upper quadrant. A ruptured spleen is diagnosed, and the client is scheduled for an emergency splenectomy. What should the nurse include when providing preoperative teaching? 1. Probability of wound dehiscence 2. Safety aspects of this type of surgery 3. Expectation of postoperative bleeding 4. Presence of abdominal drains for several days

4 Drains usually are inserted into the splenic bed to facilitate removal of fluid that can lead to abscess formation. The risk for wound dehiscence is no greater than for any other abdominal surgery. Safety aspects of this type of surgery are the role of the health care provider. Bleeding occurs more commonly with splenic repair than with removal.

A client with type 2 diabetes is taking one glyburide (Micronase) tablet daily. The client asks whether an extra pill should be taken before exercise. What is the nurse's best reply? 1. "You will need to decrease how much you are exercising." 2. "An extra pill will help your body use glucose when exercising." 3. "The amount of medication you need to take is not related to exercising." 4. "Do not take an extra pill because you may become hypoglycemic when exercising."

4 Exercise improves glucose metabolism; exercise is associated with a risk for hypoglycemia, not hyperglycemia; an additional antidiabetic agent is contraindicated. Exercise should not be decreased because it improves glucose metabolism. Also, this response does not answer the client's question. An extra tablet probably will result in hypoglycemia because exercise alone improves glucose metabolism. Control of glucose metabolism is achieved through balanced diet, exercise, and pharmacologic therapy.

The nurse is caring for a client hospitalized with a severe myocardial infarction. Which analgesic is the drug of choice for this client? 1. Diazepam (Valium) 2. Meperidine (Demerol) 3. Flurazepam (Dalmane) 4. Morphine sulfate (MS Contin)

4 For a severe myocardial infarction, morphine sulfate is the drug of choice because it relieves pain quickly and reduces anxiety. Diazepam is a muscle relaxant that may be used for its sedative effect; it is not effective for the pain of a myocardial infarction. Although meperidine iseffective, it is not the drug of choice. Flurazepam is a hypnotic that may be used to reduce fear and restlessness; it is not effective for the pain of a myocardial infarction.

A client has been experiencing chest discomfort for several months, especially after eating, which the client attributed to a hiatal hernia. The client reports that after eating a meal today, the client coughed severely and then vomited profusely. The client stated that because the vomitus was bright red in color, medical care was sought. The nurse reviews the plan of care and determines that the priority is to: 1. Begin gastric lavage 2. Obtain stool for occult blood 3. Ascertain the client's eating habits 4. Draw blood for typing and crossmatching

4 Immediate blood replacement is indicated. A type and crossmatch will ensure that the correct blood type will be administered to the client, preventing a transfusion reaction. Beginning gastric lavage is not the priority, although it may be done later. Obtaining a stool for an occult blood test is not the priority, although it may be done later. Ascertaining the client's eating habits is not the priority, although it may be done later when completing an admission history and physical.

A client who recently was diagnosed as having myelocytic leukemia discusses the diagnosis by referring to statistics, facts, and figures. The nurse determines that the client is using the defense mechanism known as: 1. Projection 2. Sublimation 3. Identification 4. Intellectualization

4 Intellectualization is the use of reasoning and thought processes to avoid the emotional aspects of a situation; this is a defense against anxiety. Projection is denying unacceptable traits and regarding them as belonging to another person. Sublimation is a defense wherein the person redirects the energy of unacceptable impulses into socially acceptable behaviors or activities. Identification is the reduction of anxiety by imitating someone respected or feared.

A client who has been diagnosed with a myocardial infarction is receiving morphine for pain. The client takes digoxin (Lanoxin) and fluoxetine (Prozac) at home. The client asks why a stool softener has been added to the medication regimen. The nurse explains the importance of not straining when having a bowel movement and that what has put the client at risk for constipation? 1. Digoxin 2. Poor peripheral perfusion 3. Fluoxetine 4. Opioid use

4 Opioids decrease peristalsis, which may precipitate constipation; straining at stool should be avoided to prevent the Valsalva maneuver, which increases demands on the heart. Digoxin is unrelated to intestinal peristalsis and the potential for constipation. Having poor peripheral perfusion is unrelated to constipation; in addition, there is no indication the client has poor peripheral circulation. A side effect of fluoxetine is diarrhea, not constipation.

Clients are encouraged to perform deep-breathing exercises after most types of surgery. The nurse teaches clients that the reason for these exercises is to help: 1. Stimulate red blood cell production 2. Expand the residual volume in the lungs 3. Decrease partial pressure of oxygen in the blood 4. Prevent the buildup of carbon dioxide in the body

4 Retention of carbon dioxide in the blood lowers the pH, causing respiratory acidosis; deep breathing maximizes gaseous exchange, ridding the body of excess carbon dioxide. Deep breathing improves oxygenation of the blood, but it does not stimulate red blood cell production. Although regular deep breathing improves the vital capacity of the lungs, residual volume is unaffected. Deep breathing increases, not decreases, the partial pressure of oxygen.

During the evening after a paracentesis, the nurse identifies that the client, although denying any discomfort, is very anxious. The best nursing approach is to: 1. Offer the client a back rub 2. Administer the prescribed opioid 3. Reinforce the primary health care provider's explanation of the procedure 4. Explore the client's concerns while administering the prescribed anxiolytic

4 Sharing and discussing concerns often release anxieties; giving the prescribed anxiolytic will produce relaxation. Offering the client a back rub might relax the client but will do little to reduce the client's level of anxiety. The client is not in pain at this time but needs to share concerns. The procedure is over; this might be appropriate before the paracentesis. Also, there are no data to support that this is the client's concern.

A client has untreated stage 1 hypertension. What is the minimum systolic pressure the nurse expects when obtaining this client's blood pressure? 1. 110 to 119 mm Hg 2. 120 to 129 mm Hg 3. 130 to 139 mm Hg 4. 140 to 159 mm Hg

4 Systolic blood pressure associated with stage 1 hypertension is between 140 and 159 mm Hg. Optimal systolic blood pressure is less than 120 mm Hg. With prehypertension, the systolic blood pressure is between 120 and 139 mm Hg. With prehypertension, the systolic blood pressure is between 120 and 139 mm Hg.

A nurse assesses a client for orthostatic hypotension. The results are: Lying - heart rate = 70 beats/minute B/P - 110/70 Sitting - heart rate = 78 beats/minute B/P - 106/66 Standing - heart rate = 85 beats/minute B/P - 100/64 The nurse would expect which primary health care provider prescription?A nurse assesses a client for orthostatic hypotension. The results are: Lying - heart rate = 70 beats/minute B/P - 110/70 Sitting - heart rate = 78 beats/minute B/P - 106/66 Standing - heart rate = 85 beats/minute B/P - 100/64 The nurse would expect which primary health care provider prescription? 1. Increase furosemide (Lasix) from 20 mg by mouth (PO) to 40 mg PO daily. 2. Give 1 L of 0.9% normal saline (NS) bolus over 4 hours. 3. Start an IV of D5 ½ NS to run at 150 mL/hr. 4. No prescription change.

4 The assessment findings do not indicate postural hypotension. There is no indication from the data that a prescription change is needed for this client. Increasing the furosemide or giving IV fluid to this client could result in a fluid imbalance in this client.

A nurse is caring for a client who was admitted with failure to thrive and severe muscle wasting due to malnutrition and immobility. When the nurse administers pills to the client, the client is not capable of holding the medicine cup. What technique should the nurse utilize to facilitate Physiological Aspects of Care? 1. Contact the client's health care provider. Ask to substitute a liquid form of medication for the pill form. 2. Remove one pill at a time from the medicine cup and place it into the client's mouth. 3. Crush the pills and administer with applesauce or ice cream. 4. Ask the client how many pills the client wants to take at one time. Drop them from the medication cup directly into the client's mouth.

4 The client needs a physical assistance with Physiological Aspects of Care and therefore the nurse should help get the pills into the client's mouth while maintaining medical aseptic technique by using the medication cup and prevent aspiration by asking client how many pills the client would like to take at once. The client does not have difficulty swallowing and therefore substitution of pills by the liquid form of medication or crushing of pills is not necessary. Putting pills into the client's mouth by the hand is not safe for the nurse and violates medical asepsis; the choice does not mention that the nurse has gloves on.

A nurse is caring for a client with a pneumothorax who has a chest tube in place with a closed drainage system. Which of these actions by the nurse is correct? 1. Strip the chest tube periodically. 2. Administer the prescribed cough suppressant at the scheduled times. 3. Empty and measure the drainage in the collection chamber each shift. 4. Keep the drainage system lower than the level of the client's chest.

4 The drainage system is kept below the chest to allow gravity to drain the pleural space. The chest tube should not be stripped because this action can cause negative pressure and damage lung tissue. Cough suppressants are not indicated because coughing and deep breathing are encouraged to help reexpand the lung. The closed system is not entered for emptying; when full, the entire device is replaced.

Immediately after a storm has passed, the nurse is working with a rescue team that is searching for injured people. The nurse finds a victim lying next to a broken natural gas main. The victim is not breathing and is bleeding heavily from a wound on the foot. What should be the nurse's first intervention? 1. Treat the victim for shock. 2. Start rescue breathing immediately. 3. Apply surface pressure to the foot wound. 4. Safely remove the victim from the immediate vicinity.

4 The first action should be to safely remove the victim from the source of further injury. Treating the victim for shock is not the priority. Breathing is the priority once further injury is avoided. Applying surface pressure to the foot wound should be the last concern. The guidelines for CPR should be followed.

A client is admitted to the hospital for a total hip replacement. Included in the health care provider's prescriptions is a prescription for digoxin (Lanoxin) 2.5 mg by mouth daily. The nurse knows that digoxin is supplied in 0.125 mg tablets. What should the nurse do? 1. Give ½ tablet. 2. Administer two tablets. 3. Ask the client what dose was taken at home. 4. Verify the prescription with the health care provider.

4 The usual dose of digoxin is 0.125 mg to 0.25 mg daily. A dose of 2.5 mg is excessive, and the prescription should be questioned. Half a tablet is not the prescribed dose; the prescribed dose is excessive and must be questioned. Two tablets are not the prescribed dose; the prescribed dose is excessive and must be questioned. Asking the client what dose was taken at home might be done, although it is not as important as verifying the prescription. The nurse should not administer medication that is outside therapeutic parameters.

A nurse expects that a client with right-sided heart failure will exhibit: 1. Oliguria 2. Pallor 3. Cool extremities 4. Distended neck veins

4 Veins are distended because of the systemic venous pressure and congestion that are associated with right-sided heart failure. Oliguria, pallor, and cool extremities are key features of left-sided heart failure.


Kaugnay na mga set ng pag-aaral

Chapter 1 Vocabulary - Ratios and Proportional Reasoning

View Set

Life Insurance Chapter 3 Life Insurance Basics

View Set

Chapter 23: Pediatric Nursing Interventions and Skills

View Set

Phong trào dân tộc dân chủ ở việt nam từ năm 1925 đến năm 1930

View Set